Крок 1 - Медицина 2005 (буклет)

1 / 200
До хірургічного відділення надійшов чоловік 35 років з гнійною раною шиї попереду трахеї (у ділянці передвісцерального простору). Куди може поширитися інфекція, якщо терміново не буде проведене оперативне втручання? A 35-year-old man was admitted to the surgical department with a purulent neck wound in front of the trachea (in the area of the previsceral space). Where can the infection spread if urgent surgical intervention is not performed?

У ретровісцеральний простір Into retrovisceral space

У заднє середостіння Into the posterior mediastinum

У переднє середостіння In the anterior mediastinum

У надгруднинний міжапоневротичний простір Into the suprasternal interaponeurotic space

У середнє середостіння In the middle mediastinum

2 / 200
У хворого з гіпохромною анемією січеться та випадає волосся, відзначається підвищена ламкість нігтів, порушений смак. Якою є причина розвитку зазначених симптомів? A patient with hypochromic anemia has thinning and hair loss, increased brittleness of nails, impaired taste. What is the reason for the development of these symptoms?

Дефіцит вітаміну А Vitamin A deficiency

Зниження продукції паратирину Decreasing production of parathyrin

Дефіцит вітаміну В12 Vitamin B12 deficiency

Дефіцит залізовмісних ферментів Deficiency of iron-containing enzymes

Зниження продукції тиреоїдних гормонів Decreased production of thyroid hormones

3 / 200
До медико-генетичної консультації звернулася жінка з приводу ризику захворювання на гемофілію у свого сина. Її чоловік страждає на це захворювання від народження. Жінка та її родичі не страждали на це захворювання. Визначте вірогідність народження хлопчика з гемофілією у цій сім’ї: A woman turned to medical and genetic counseling regarding the risk of hemophilia in her son. Her husband has suffered from this disease since birth. The woman and her relatives did not suffer from this is a disease. Determine the probability of a boy with hemophilia being born in this family:

75% хлопчиків будуть хворими 75% of boys will be sick

100% 100%

50% хлопчиків будуть хворими 50% of boys will be sick

25% хлопчиків будуть хворими 25% of boys will be sick

0% 0%

4 / 200
Послаблення кровопостачання органа зумовлює розвиток гіпоксії, яка активізує функцію фібробластів. Об’єм яких елементів збільшується в цій ситуації? Weakening of the organ's blood supply leads to the development of hypoxia, which activates the function of fibroblasts. The volume of which elements increases in this situation?

Судини мікроциркуляторного русла Vessels of the microcirculatory channel

Нервові елементи Nervous elements

Лімфатичні судини Lymphatic vessels

Паренхіматозні елементи органа Parenchymatous elements of the organ

Міжклітинна речовина Intercellular substance

5 / 200
При мікроскопії зіскобу з періанальних складок виявлені безбарвні яйця, що мають форму несиметричних овалів, розміром 50x23 мкм. Про який вид гельмінту йде мова? When microscopically scraping from the perianal folds, colorless eggs with the shape of asymmetric ovals, 50x23 microns in size, were found. What kind of helminth is we talking about?

Карликовий ціп’як Dwarf creeper

Кривоголовка Curvyhead

Аскарида Ascarida

Гострик Hostrik

Волосоголовець Hairhead

6 / 200
У здорового обстежуваного в стані спокою кількість еритроцитів становить 5,65*1012/л. Причиною цього може бути те, що обстежуваний: The number of erythrocytes in a healthy subject at rest is 5.65*1012/l. The reason for this may be that the subject :

Вагітна жінка Pregnant woman

Відповідальний працівник міністерства Responsible employee of the ministry

Мешканець високогір’я Highlander

Студент Student

Шахтар Miner

7 / 200
До лікаря звернулася мати, син якої за літо виріс на 18 см. При обстеженні хлопця 12 років: зріст - 180 см, вага 68 кг. З гіперфункцією якої ендокринної залози це може бути пов’язано? A mother went to the doctor, whose son grew 18 cm over the summer. During the examination of the 12-year-old boy: height - 180 cm, weight 68 kg. With hyperfunction of which endocrine gland it can be connected?

Епіфіз Pineal gland

Наднирник Adrenal gland

Гіпофіз Pituitary

Щитоподібна Thyroid

Підшлункова Pancreatic

8 / 200
Хворий не розуміє змісту слів, а також не розуміє власної мови (словесна глухота). Яка із структур кори великих півкуль головного мозку уражена? The patient does not understand the meaning of words, and also does not understand his own language (verbal deafness). Which of the structures of the cortex of the large hemispheres of the brain is affected?

Нижня лобова звивина Inferior frontal gyrus

Верхня тім’яна часточка Superior parietal lobe

Нижня тім’яна часточка Inferior parietal lobe

Задня центральна звивина Posterior central gyrus

Верхня скронева звивина Superior temporal gyrus

9 / 200
Територію старого худобомогильника, який не використовувався більше 50 років, планується відвести під житлове будівництво. Однак дослідження грунту виявило наявність життєздатних спор збудника особливо небезпечного захворювання. Який із вказаних мікроорганізмів найбільш вірогідно міг зберігатися у грунті протягом такого тривалого часу? The territory of the old livestock burial ground, which has not been used for more than 50 years, is planned to be used for residential construction. However, soil research revealed the presence of viable spores of the causative agent of a particularly dangerous disease. Which of the following microorganisms could most likely be stored in the soil for such a long time?

Brucella abortus Brucella abortus

Frantisella tularensis Frantisella tularensis

Batillus anthratis Batillus anthratis

Mycobacterium bovis Mycobacterium bovis

Yerarna pestis Yerarna pestis

10 / 200
Молода людина під час активного підтягування на перекладині відчула різкий біль у спині. Об’єктивно: біль при спробах руху верхньою кінцівкою, обмеження функцій приведення та пронації. Розтягнення якого м’яза, найбільш вірогідно, відбулося? A young man during an active pull-up on the crossbar felt a sharp pain in his back. Objectively: pain when trying to move the upper limb, limitation of adduction and pronation functions. Stretching of which muscle, most likely occurred?

М.romboideus major M.romboideus major

М.subscapularis M.subscapularis

M.trapezrns M.trapezrns

М.levator scapulae M.levator scapulae

М.latissimus dorsi M.latissimus dorsi

11 / 200
Людина хворіє на цукровий дiабет, що супроводжується гіперглікемією натще понад 7,2 ммоль/л. Рівень якого білка плазми крові дозволяє ретроспективно (за попередні 4-8 тижні до обстеження) оцінити рівень глікемії? A person has diabetes mellitus accompanied by fasting hyperglycemia over 7.2 mmol/l. The level of which blood plasma protein allows retrospectively (for the previous 4-8 weeks before examination) to assess the level of glycemia?

Ілікозильований гемоглобін Ilicosylated hemoglobin

С-реактивний білок C-reactive protein

Альбумін Albumin

Церулоплазмін Ceruloplasmin

Фібриноген Fibrinogen

12 / 200
У хворого із значними периферічними набряками почергове застосування дихлотіазиду, етакринової кислоти та фуросеміду не викликало значного діуретичного ефекту. У крові значне підвищення кількості альдостерону. Вкажіть препарат вибору: In a patient with significant peripheral edema, alternate use of dichlothiazid, ethacrynic acid and furosemide did not cause a significant diuretic effect. Aldosterone was significantly increased in the blood. Specify the drug of choice:

Сечовина Urea

Амілорид Amiloride

Спіронолактон Spironolactone

Маніт Manit

Клопамід Clopamide

13 / 200
Під час лапаротомії хірург виявив гангренозне ураження низхідної ободової кишки. Тромбоз якої артерії зумовив цей стан? During laparotomy, the surgeon found a gangrenous lesion of the descending colon. Thrombosis of which artery caused this condition?

Верхня брижова Upper Mesentery

Клубово-ободова Club-rim

Серединна ободова Middle Rim

Ліва ободова Left rim

Права ободова Right Rim

14 / 200
Під час розтину тіла померлої в комі молодої людини, виявлено: поширений тромбоемболічний інфаркт лівої півкулі мозку, велика септична селезінка, імунокомплексний гломерулонефрит, виразки на стулках аортального клапану, що прикриті поліпоподібними тромбами з колоніями стафілококів. Яке захворювання викликало церебральну тромбоемболію? During the autopsy of the body of a young man who died in a coma, the following were found: widespread thromboembolic infarction of the left hemisphere of the brain, a large septic spleen, immune complex glomerulonephritis, ulcers on the leaflets of the aortic valve, which covered by polyp-like thrombi with colonies of staphylococci. What disease caused cerebral thromboembolism?

Септичний бактеріальний ендокардит Septic bacterial endocarditis

Ревматичний тромбендокардит Rheumatic thromboendocarditis

Септикопіємія Septicopyemia

Септицемія Septicemia

Гострий ревматичний вальвуліт Acute rheumatic valvulitis

15 / 200
Внаслідок дії електричного струму на збудливу клітину виникла деполяризація її мембрани. Рух яких іонів через мембрану є причиною деполяризації? As a result of the action of an electric current on an excitable cell, its membrane depolarized. The movement of which ions through the membrane is the cause of depolarization?

Cl- Cl-

НСО НСО

Na+ Na+

Са2+ Ca2+

K+ K+

16 / 200
Постраждалому з колотою раною передньої стінки шлунка надається хірургічна допомога. До якого утворення порожнини очеревини потрапив вміст шлунка? Surgical assistance is provided to a victim with a stab wound of the front wall of the stomach. What formation of the peritoneal cavity did the contents of the stomach enter?

Передшлункова сумка Pregastric pouch

Правий мезентеріальний синус Right mesenteric sinus

Сальникова сумка Gland bag

Печінкова сумка Liver bag

Лівий мезентеріальний синус Left mesenteric sinus

17 / 200
Вагітній жінці, що мала в анамнезі декілька викиднів, призначена терапія, яка містить вітамінні препарати. Укажіть вітамін, який сприяє виношуванню вагітності: A pregnant woman with a history of multiple miscarriages is prescribed therapy that includes vitamin preparations. Specify the vitamin that promotes pregnancy:

Піридоксальфосфат Pyridoxal phosphate

Рутін Routine

Цианкобаламін Cyanocobalamin

α-токоферол α-tocopherol

Фолієва кислота Folic acid

18 / 200
У хворого з аневризмою правої підключичної артерії спостерігається осиплість голосу. З подразненням якого нерву це може бути пов’язане? A patient with an aneurysm of the right subclavian artery has a hoarseness of voice. What nerve irritation can this be associated with?

N.laringeus superior dexter N.laryngeus superior dexter

N.laringeus reccurens srnlster N.laryngeus reccurens srnlster

N.laringeus inferior smster N.laryngeus inferior smster

N.laringeus reccurens dexter N.laryngeus reccurens dexter

N.laringeus superior smster N.laryngeus superior smster

19 / 200
До приймального відділення доставлений хворий зі скаргами на сухість у роті, світлобоязнь та порушення зору. Шкіра гіперемована, суха, зіниці розширені, тахікардія. При подальшому обстеженні був встановлений діагноз: отруєння алкалоїдами красавки. Який із лікарських засобів доцільно застосувати? A patient was brought to the reception department with complaints of dry mouth, photophobia, and impaired vision. The skin is hyperemic, dry, dilated pupils, tachycardia. Upon further examination, the diagnosis was established : poisoning by alkaloids of a beauty. Which of the medicines should be used?

Прозерин Proserin

Дипіроксим Dipiroxime

Діазепам Diazepam

Пілокарпін Pilocarpine

Армін Armin

20 / 200
У пацієнта після травми виникли паралічі, розлади дотикової чутливості з правого боку; зліва паралічі відсутні, але порушена больова та температурна чутливість. Яка причина цього явища? After the injury, the patient developed paralysis, disorders of touch sensitivity on the right side; on the left side, there is no paralysis, but the pain and temperature sensitivity is disturbed. What is the cause of this phenomenon?

Однобічне пошкодження спинного мозку з правого боку Unilateral spinal cord injury on the right side

Пошкодження середнього мозку Midbrain damage

Пошкодження мозочка Cerebellar damage

Пошкодження стовбура мозку Brain stem damage

Пошкодження рухової зони кори головного мозку Damage of the motor cortex of the brain

21 / 200
Оглядаючи черевну порожнину потерпілого з проникним пораненням передньої черевної стінки, хірург помітив пошкодження малого сальника, з якого витікала яскраво-червона (артеріальна) кров. Яка артерія виявилася пошкодженою? While examining the abdominal cavity of a victim with a penetrating wound of the anterior abdominal wall, the surgeon noticed damage to the small omentum, from which bright red (arterial) blood flowed. Which artery was damaged?

A.lienalis A.lienalis

A.gastroepiploica dextra A.gastroepiploica dextra

A.hepatica communis A.hepatica communis

A.hepatica propria A.hepatica propria

A.gastroepiploica sinistra A.gastroepiploica sinistra

22 / 200
У пологовому будинку народилася дитина з численними порушеннями як зовнішніх так і внутрішніх органів - серця, нирок, травної системи. Був встановлений попередній діагноз - синдром Дауна. За допомогою якого методу можна підтвердити цей діагноз? A child was born in the maternity hospital with numerous disorders of both external and internal organs - heart, kidneys, digestive system. A preliminary diagnosis was established - Down's syndrome. With the help of method can this diagnosis be confirmed?

Біохімічний Biochemical

Популяційно-статистичний Population-statistical

Генеалогічний Genealogical

Близнюковий Twin

Цитогенетичний Cytogenetic

23 / 200
У процесі метаболізму в організмі людини утворюються активні форми кисню, у тому числі супероксидний аніонрадикал. За допомогою якого ферменту інактивується цей аніон? In the process of metabolism in the human body, active forms of oxygen are formed, including the superoxide anion radical. Which enzyme inactivates this anion?

Каталаза Catalase

Пероксидаза Peroxidase

Глутатіонпероксидаза Glutathione peroxidase

Супероксиддисмутаза Superoxide dismutase

Глутатіонредуктаза Glutathione reductase

24 / 200
Експериментатору необхідно якнайшвидше виробити умовний рефлекс у собаки. На базі якого безумовного рефлексу доцільно виробляти умовний рефлекс у цьому випадку? The experimenter needs to produce a conditioned reflex in the dog as soon as possible. On the basis of which unconditioned reflex is it advisable to produce a conditioned reflex in this case?

Захисний Protective

Міостатичний Myostatic

Травний Digestible

Орієнтувальний Indicative

Статевий Gender

25 / 200
Студент старанно конспектує лекцію. Якість конспектування значно погіршилася, коли сусіди стали розмовляти. Який вид гальмування в корі головного мозку є причиною цього? The student is diligently taking notes of the lecture. The quality of the notes deteriorated significantly when the neighbors started talking. What kind of inhibition in the cerebral cortex is the reason for this?

Запізніле Late

Диференціювальне Differentiating

Згасаюче Fading

Позамежове Foreign

Зовнішнє External

26 / 200
Під час патронажу лікар виявив у дитини симетричну шорсткість щік, діарею, порушення нервової діяльності. Нестача яких харчових факторів є причиною такого стану? During the visit, the doctor found symmetrical roughness of the cheeks, diarrhea, and nervous disorders in the child. The lack of which nutritional factors is the cause of this condition?

Треонін, пантотенова кислота Threonine, pantothenic acid

Лізин, аскорбінова кислота Lysine, ascorbic acid

Фенілаланін, пангамова кислота Phenylalanine, pangamic acid

Метіонін, ліпоєва кислота Methionine, lipoic acid

Нікотинова кислота, триптофан Nicotinic acid, tryptophan

27 / 200
Для вивчення локалізації біосинтезу білка в клітинах, миші ввели мічені амінокислоти аланін та триптофан. Біля яких органел буде спостерігатися накопичення мічених амінокислот? To study the localization of protein biosynthesis in cells, mice were injected with labeled amino acids alanine and tryptophan. Near which organelles will the accumulation of labeled amino acids be observed?

Клітинний центр Cell Center

Апарат Гольджі Golgi Apparatus

Гладенька ЕПС Smooth EPS

Рибосоми Ribosomes

Лізосоми Lysosomes

28 / 200
У хворих на колагеноз має місце процес деструкції сполучної тканини. Це підтверджується збільшенням у крові: In patients with collagenosis, the process of destruction of connective tissue takes place. This is confirmed by an increase in the blood:

Вмісту креатину та креатиніну Content of creatine and creatinine

Активності трансаміназ Activities of transaminases

Вмісту уратів Urate content

Активності ізоферментів ЛДГ Activities of LDH isozymes

Вмісту оксипроліну та оксилізину Oxyproline and oxylysine content

29 / 200
У 6-місячної дитини спостерігалися часті та інтенсивні підшкірні крововиливи. Призначення синтетичного аналога вітаміну K (вікасолу) дало позитивний ефект. У укарбоксилюванні глутамінової кислоти якого білка зсідальної системи крові бере участь цей вітамін? Frequent and intense subcutaneous hemorrhages were observed in a 6-month-old child. The appointment of a synthetic analogue of vitamin K (Vikasol) had a positive effect. In the carboxylation of glutamic acid, which protein of the blood-forming system is this vitamin involved?

Фібриноген Fibrinogen

Протромбін Prothrombin

Фактор Хагемана Hageman Factor

Антигемофільний глобулін A Antihemophilic globulin A

Фактор Розенталя Rosenthal Factor

30 / 200
У хворого 60 років виявлено розширення вен стравоходу, прямої кишки та підшкірних вен передньої черевної стінки. Система якої вени ушкоджена? A 60-year-old patient was found to have enlarged veins of the esophagus, rectum, and subcutaneous veins of the anterior abdominal wall. Which vein system is damaged?

Верхня брижова Upper Mesentery

Верхня порожниста Top hollow

Нижня порожниста Bottom Hollow

Ворітна Voritna

Непарна Odd

31 / 200
У хворого лікар діагностував гостру гонорею. З анамнезу відомо, що раніше він переніс гонорею та вилікування було повним. До якої категорії інфекцій можна віднести це нове захворювання? The doctor diagnosed the patient with acute gonorrhea. It is known from the anamnesis that he previously suffered from gonorrhea and the cure was complete. To which category of infections can this new disease be classified?

Суперінфекція Superinfection

Рецидив Relapse

Реінфекція Reinfection

Вторинна інфекція Secondary infection

Аутоінфекція Auto-infection

32 / 200
У людини зменшений діурез, гіпернатріємія, гіпокаліємія. Гіперсекреція якого гормону може бути причиною таких змін? A person has reduced diuresis, hypernatremia, hypokalemia. Hypersecretion of which hormone can cause such changes?

Передсердний натрійуретичний фактор Atrial natriuretic factor

Вазопресин Vasopressin

Паратгормон Parathyroid hormone

Адреналін Adrenaline

Альдостерон Aldosterone

33 / 200
Під час вивчення фаз мітотичного циклу корінця цибулі знайдено клітину, в якій хромосоми лежать в екваторіальній площині, утворюючи зірку. На якій стадії мітозу перебуває клітина? While studying the phases of the mitotic cycle of an onion root, a cell was found in which the chromosomes lie in the equatorial plane, forming a star. At what stage of mitosis is the cell?

Метафаза Metaphase

Анафаза Anaphase

Профаза Prophase

Інтерфаза Interphase

Телофаза Telophase

34 / 200
У хворого напад тахікардії. Які мембранні циторецептори кардіоміоцитів доцільно заблокувати, щоб припинити напад? The patient has a tachycardia attack. Which membrane cytoreceptors of cardiomyocytes should be blocked to stop the attack?

в-адренорецептори v-adrenoceptors

Н-холінорецептори H-cholinergic receptors

М- та Н-холінорецептори M- and H-cholinergic receptors

а-адренорецептори α-adrenoceptors

М-холінорецептори M-cholinergic receptors

35 / 200
У крові хворого концентрація альбумінів складає 2,8 г/л, підвищена концентрація лактатдегідрогенази 5 (ЛДГ5). Про захворювання якого органа це свідчить? The concentration of albumin in the patient's blood is 2.8 g/l, the concentration of lactate dehydrogenase 5 (LDH5) is elevated. Which organ disease does this indicate?

Серце Heart

Легеня Lung

Нирка Kidney

Печінка Liver

Селезінка Spleen

36 / 200
Під час розтину тіла мертвонародженої дитини виявлено аномалію розвитку серця: шлуночки не розмежовані, з правої частини виходить суцільний артеріальний стовбур. Для якого класу хребетних тварин характерна подібна будова серця? During the autopsy of the body of a stillborn child, an abnormality in the development of the heart was found: the ventricles are not demarcated, and a continuous arterial trunk emerges from the right part. What class of vertebrate animals is characterized by a similar structure of the heart?'

Ссавці Mammals

Рептилії Reptiles

Амфібії Amphibia

Риби Fish

Птахи Birds

37 / 200
У чоловіка 59 років спострерігаються ознаки паренхіматозної жовтяниці та портальної гіпертензії. Під час гістологічного дослідження пункційного біоптату печінки знайдено: балково-часточкова будова порушена, частина гепатоцитів має ознаки жирової дистрофії, утворюються порто-портальні сполучнотканинні септи з формуванням псевдочасточок, з наявністю пері-портальних лімфомакрофагальних інфільтратів. Який найбільш вірогідний діагноз? A 59-year-old man has signs of parenchymal jaundice and portal hypertension. During histological examination of a puncture biopsy of the liver, it was found: the beam-lobular structure is disturbed, part of the hepatocytes has signs of fatty dystrophy , porto-portal connective tissue septa are formed with the formation of pseudoparticles, with the presence of peri-portal lympho-macrophagic infiltrates. What is the most likely diagnosis?

Хронічний гепатоз Chronic hepatosis

Токсична дистрофія Toxic dystrophy

Вірусний гепатит Viral hepatitis

Алкогольний гепатит Alcoholic hepatitis

Цироз печінки Liver cirrhosis

38 / 200
До гастроентерологічного відділення надійшов хворий із запаленням жовчних шляхів. У порціях жовчі виявлено рухомі найпростіші грушоподібної форми, двоядерні, з опорним стрижнем - аксостилем. Яке протозойне захворювання діагностується у хворого? A patient with inflammation of the biliary tract was admitted to the gastroenterology department. In portions of the bile, motile pear-shaped protozoa, binucleate, with a supporting rod - an axostyle, were found. What protozoan disease is diagnosed in the patient ?

Балантидіаз кишковий Intestinal balantidiasis

Амебна дизєнтєрія Amoebic dysentery

Лямбліоз Giardiasis

Амебіаз кишковий Intestinal amebiasis

Трихомоноз Trichomoniasis

39 / 200
У хворого 30 років з рiзаною раною передпліччя виникло порушення розгинання пальців кисті. Про пошкодження якого нерва це свідчить? A 30-year-old patient with a cut wound on the forearm developed a violation of the extension of the fingers of the hand. What nerve damage does this indicate?

Променевий Radial

Серединний Middle

Ліктьовий Cubit

Присередній шкірний нерв передпліччя Median cutaneous nerve of the forearm

М’язовошкірний Musculocutaneous

40 / 200
Під час обробки атипових кардіо-міоцитів синоатріального вузла біологічно активною речовиною, зареєстровано збільшення їх мембранного потенціалу через збільшену проникність для іонів калію. Яка біологічно активна речовина впливала на кардіоміоцити? During the treatment of atypical cardiomyocytes of the sinoatrial node with a biologically active substance, an increase in their membrane potential was recorded due to increased permeability to potassium ions. What biologically active substance affected cardiomyocytes?'

Тироксин Thyroxine

Ацетилхолін Acetylcholine

Норадреналін Noradrenaline

Атріопептид Atriopeptide

Адреналін Adrenaline

41 / 200
Під час експерименту подразнюють скелетний м’яз серією електричних імпульсів. Який вид м’язового скорочення буде виникати, якщо кожний наступний імпульс надходить у періоді вкорочення попереднього поодинокого м’язового скорочення? During the experiment, a skeletal muscle is stimulated with a series of electrical impulses. What type of muscle contraction will occur if each subsequent impulse arrives during the period of shortening of the previous single muscle' ulnar reduction?

Асинхронний тетанус Asynchronous tetanus

Зубчастий тетанус Serrated tetanus

Контрактура м’яза Muscle contracture

Суцільний тетанус Solid tetanus

Серія поодиноких скорочень Series of single abbreviations

42 / 200
У хворого через декілька годин після опіку в ділянці гіперемії та набряку шкіри з’явилося вогнище некрозу. Який головний механізм забезпечує посилення руйнівних явищ в осередку запалення? In the patient, several hours after the burn, a focus of necrosis appeared in the area of hyperemia and swelling of the skin. What is the main mechanism that ensures the increase of destructive phenomena in the center of inflammation?

Проліферація фібробластів Proliferation of fibroblasts

Вторинна альтерація Secondary alteration

Еміграція лімфоцитів Emigration of lymphocytes

Діапедез еритроцитів Diapedesis of erythrocytes

Первинна альтерація Primary alteration

43 / 200
У чоловіка 42 років, який страждає на подагру, в крові підвищена концентрація сечової кислоти. Для зниження рівня сечової кислоти йому призначено алопуринол. Конкурентним інгібітором якого ферменту є алопуринол? A 42-year-old man suffering from gout has an increased concentration of uric acid in his blood. He is prescribed allopurinol to reduce the level of uric acid. Allopurinol is a competitive inhibitor of which enzyme?'

Аденозиндезаміназа Adenosine deaminase

Гуаніндезаміназа Guanindeaminase

Ксантиноксидаза Xanthine oxidase

Аденінфосфорибозилтрансфераза Adenine phosphoribosyltransferase

Гіпоксантинфосфорибозилтрансфераза Hypoxanthine phosphoribosyltransferase

44 / 200
Хлопчик 13 років скаржиться на загальну слабкість, запаморочення, втомлюваність. Спостерігається відставання у розумовому розвитку. При обстеженні виявлено високу концентрацію валіну, ізолейцину, лейцину в крові та сечі. Сеча специфічного запаху. Який найбільш вірогідний діагноз? A 13-year-old boy complains of general weakness, dizziness, fatigue. There is a delay in mental development. The examination revealed a high concentration of valine, isoleucine, leucine in the blood and urine. Urine with a specific smell. What is the most likely diagnosis?

Хвороба 'кленового сиропу' Maple syrup disease

Хвороба Аддісона Addison's disease

Тирозиноз Tyrosinosis

Базедова хвороба Bazed disease

Гістидинемія Histidinemia

45 / 200
Хворий звернувся із скаргами на загальну слабкість, головний біль, нудоту, блювання, рідкі випорожнення з домішками слизу та крові. При мікроскопії дуоденального вмісту та при дослідженні свіжих фекалій виявлено рухомі личинки. Який найбільш вірогідний діагноз? The patient complained of general weakness, headache, nausea, vomiting, loose stools with impurities of mucus and blood. Microscopy of duodenal contents and examination of fresh feces revealed moving larvae. What is the most likely diagnosis?

Дракункульоз Dracunculosis

Анкілостомідоз Ankylostomidosis

Трихоцефальоз Trichocephalus

Стронгілоїдоз Strongyloidosis

Ентеробіоз Enterobiosis

46 / 200
Біля інфікованої рани збільшилися регіонарні лімфовузли. Під час гістологічного дослідження в них виявлено збільшення кількості макрофагів, лімфоцитів та лімфатичних фолікулів у кірковому шарі, а також велику кількість плазматичних клітин. Який процес відображують виявлені у лімфатичних вузлах гістологічні зміни? Near the infected wound, the regional lymph nodes increased. During histological examination, an increase in the number of macrophages, lymphocytes, and lymphatic follicles in the cortical layer, as well as a large number of plasma cells, was found in them. What process do the histological changes detected in the lymph nodes reflect?

Антигенна стимуляція Antigenic stimulation

Набута недостатність лімфоїдної тканини Acquired insufficiency of lymphoid tissue

Природжена недостатність лімфоїдної тканини Congenital insufficiency of lymphoid tissue

Реакція гіперчутливості Hypersensitivity reaction

Пухлинна трансформація Tumor transformation

47 / 200
Жінка 62 років скаржиться на частий біль у ділянці грудної клітки та хребта, переломи ребер. Лікар припустив мієломну хворобу (плазмоцитому). Який з перерахованих нижче лабораторних показників буде мати найбільше діагностичне значення? A 62-year-old woman complains of frequent chest and spine pain, rib fractures. The doctor suspected myeloma (plasmocytoma). Which of the following laboratory indicators will she have the greatest diagnostic value?

Гiперальбумiнемiя Hyperalbuminemia

Гіпопротеїнемія Hypoproteinemia

Протєїнурія Proteinuria

Парапротеїнемiя Paraproteinemia

Гіпогло6улінємія Hypoglobulinemia

48 / 200
У гістопрепараті представлено паренхіматозний орган, що має кіркову та мозкову речовину. Кіркова речовина утворена тяжами епітеліоцитів, між якими проходять кровоносні капіляри; тяжі формують три зони. Мозкова речовина складається з хромафіноцитів та венозних синусоїдів. Який орган має зазначені морфологічні ознаки? The histopreparation shows a parenchymal organ with cortical and medullary substance. The cortical substance is formed by strands of epitheliocytes, between which blood capillaries pass; the strands form three zones. The medulla consists of from chromaffinocytes and venous sinusoids. Which organ has the specified morphological features?

Наднирник Adrenal gland

Щитовидна залоза Thyroid

Тимус Thymus

Лімфатичний вузол Lymph node

Нирка Kidney

49 / 200
Хворий надійшов зі скаргами на диспептичні розлади, мелену, гемороїдальні кровотечі. Під час дослідження виявлено розширення сітки венозних судин на передній стінці живота, збільшення його розмірів. Яка патологія може мати такі симптоми? The patient came in with complaints of dyspeptic disorders, melena, hemorrhoidal bleeding. During the examination, an expansion of the network of venous vessels on the front wall of the abdomen, an increase in its size, was revealed. What pathology can have such symptoms?

Коліт Colitis

Ентерит Enteritis

Кишкова аутоінтоксикація Intestinal autointoxication

Портальна гіпертензія Portal hypertension

Виразкова хвороба Ulcer disease

50 / 200
Хворому, що страждає на склеродермію (колагенове захворювання), було призначено імунодепресант, який відноситься до групи протималярійних засобів, похідних хіноліну. Хворий приймав препарат тривало. Внаслідок цього у нього знизилася вага, посивіло волосся, зменшилася гострота зору, з’явилося миготіння перед очима. Було призначено обстеження у окуліста, який відмінив препарат. Визначте, яким препаратом лікували хворого: A patient suffering from scleroderma (collagen disease) was prescribed an immunosuppressant, which belongs to the group of antimalarial drugs, quinoline derivatives. The patient took the drug for a long time. As a result, in his weight decreased, his hair turned gray, his visual acuity decreased, flickering appeared before his eyes. An examination by an ophthalmologist was scheduled, who canceled the drug. Determine which drug the patient was treated with:

Циклофосфан Cyclophosphane

Хінгамін Hingamine

Кризанол Cryzanol

Метотрексат Methotrexate

Хіноцид Khinocide

51 / 200
У загальному вигляді генетичний апарат еукаріот є таким: ек-зон-інтрон-екзон. Така структурно-функціональна організація гена зумовлює особливості транскрипції. Якою буде про-і-РНК відповідно до згаданої схеми? In general, the genetic apparatus of eukaryotes is as follows: exon-zone-intron-exon. This structural and functional organization of the gene determines the features of transcription. What will be the pro-and- RNA according to the mentioned scheme?

Екзон-екзон Exon-Exon

Екзон-екзон-інтрон Exon-exon-intron

Екзон-інтрон Exon-intron

Екзон-інтрон-екзон Exon-Intron-Exon

Інтрон-екзон Intron-exon

52 / 200
хворого виявлено підвищення активності ЛДГ1 ЛДГ2 52. , АсАТ, креатинкінази. В якому органі хворого найбільш вірогідний розвиток патологічного процесу? increased activity of LDH1 LDH2 52. , AST, creatine kinase was detected in the patient. In which organ of the patient is the most likely development of the pathological process?

Нирки Kidneys

Скелетні м’язи Skeletal muscles

Серце Heart

Підшлункова залоза Pancreas

Печінка Liver

53 / 200
Споживання пацієнтом протягом тривалого часу забруднених овочів та фруктів призвело до отруєння нітратами. Яке похідне гемоглобіну утворилося у крові даного хворого? Consumption of contaminated vegetables and fruits by the patient for a long time led to nitrate poisoning. What hemoglobin derivative was formed in the blood of this patient?

Hb-OH Hb-OH

Hb СО Hb CO

Hb NHCOOH Hb NHCOOH

Hb O2 Hb O2

Hb CN Hb CN

54 / 200
3 роки тому хворій 34 років було встановлено діагноз хронічного гломерулонефриту. За останні 6 місяців з’явилися набряки. Що лежить в основі їх розвитку? 3 years ago, a 34-year-old patient was diagnosed with chronic glomerulonephritis. In the last 6 months, edema appeared. What is the basis of their development?

Гіперосмолярність плазми Hyperosmolarity of plasma

Гіперпродукція вазопресину Hyperproduction of vasopressin

Гіперальдостеронізм Hyperaldosteronism

Порушення білковоутворюючої функції печінки Disruption of the protein-forming function of the liver

Протеїнурія Proteinuria

55 / 200
Під час роботи щодо ліквідації наслідків аварії на АЕС, робітник одержав дозу опромінення 500 рентген. Скаржиться на головний біль, нудоту, запаморочення. Які зміни кількості лейкоцитів можна очікувати у хворого через 10 годин після опромінення? During the work to eliminate the consequences of the accident at the nuclear power plant, the worker received a radiation dose of 500 x-rays. He complains of headache, nausea, dizziness. What changes in the number of leukocytes can be expected in the patient 10 hours after irradiation?

Лейкопенія Leukopenia

Лейкемія Leukemia

Лімфоцитоз Lymphocytosis

Агранулоцитоз Agranulocytosis

Нейтрофільний лейкоцитоз Neutrophil leukocytosis

56 / 200
З хімічного виробництва до токсикологічного відділення доставлений хворий з отруєнням парами ртуті. Який препарат слід використати в даній ситуації? A patient with mercury vapor poisoning was brought from the chemical industry to the toxicology department. What drug should be used in this situation?

Активоване вугілля Activated carbon

Ізонітрозин Isonitrosine

Унітіол Unithiol

Налоксон Naloxone

Ентеросорбент СКН Enterosorbent SKN

57 / 200
У раціоні людини велика кількість вуглеводів. Кількість яких структур збільшиться у цитоплазмі гепатоцитів? There is a large amount of carbohydrates in the human diet. What structures will increase in the cytoplasm of hepatocytes?

Гранули глікогену Glycogen granules

Включення ліпофусцину Inclusion of lipofuscin

Краплини жиру Fat Drops

Вільні рибосоми Free ribosomes

Лізосоми Lysosomes

58 / 200
Жінка 63 років має ознаки ревматоїдного артриту. Підвищення рівня якого з перерахованих нижче показників крові буде найбільш значущим для підтвердження діагнозу? A 63-year-old woman has symptoms of rheumatoid arthritis. An increase in the level of which of the following blood parameters would be most significant for confirming the diagnosis?

Ліпопротеїди Lipoproteins

Загальний холестерин Total cholesterol

Сумарні глікозаміноглікани Total glycosaminoglycans

R-глікозидаза R-glycosidase

Кисла фосфатаза Acid phosphatase

59 / 200
Під час розтину на зовнішній поверхні аортального клапана, виявлені великі (1-2 см) буровато-червоні, крихкі нашарування, які прикривають виразкові дефекти. Який найбільш вірогідний діагноз? During the autopsy on the outer surface of the aortic valve, large (1-2 cm) brownish-red, fragile layers were found, which covered ulcerative defects. What is the most likely diagnosis ?

Поліпозно-виразковий ендокардит Polyposis-ulcerative endocarditis

Гострий бородавчастий ендокардит Acute warty endocarditis

Дифузний ендокардит Diffuse endocarditis

Поворотній бородавчастий ендокардит Rotary verrucous endocarditis

Фібропластичний ендокардит Fibroplastic endocarditis

60 / 200
Після крововиливу в мозок у хворого стали неможливими активні рухи лівих руки та ноги. Тонус м’язів цих кінцівок підвищено, їх спінальні рефлекси різко підсилені, розширено зони рефлексів. Рефлекс Бабінського зліва. Який вид розладу рухів має місце у хворого? After a cerebral hemorrhage, active movements of the left arm and leg became impossible for the patient. The tone of the muscles of these limbs increased, their spinal reflexes were sharply strengthened, and the reflex zones were expanded. Babinski's reflex on the left. What type of movement disorder does the patient have?

Периферичний параліч Peripheral paralysis

В’ялий параліч Flax paralysis

Рефлекторний параліч Reflex paralysis

Спінальний шок Spinal shock

Центральний параліч Central paralysis

61 / 200
Під час рентгенологічного дослідження кісток основи черепа виявлено збільшення турецького сідла та деструкція окремих його ділянок. Пухлинне розростання якого анатомічного утворення може спричинити таке руйнування кістки? During an X-ray examination of the bones of the base of the skull, an increase in the sella turcica and the destruction of some of its areas were revealed. Tumor growth of which anatomical formation can cause such destruction of the bone?

Епіфіз Pineal gland

Гіпофіз Pituitary

Чотиригорбове тіло Four hump body

Колінчасті тіла Claniform bodies

Зоровий горб Visual hump

62 / 200
Хворий відзначає часті проноси, особливо після вживанняжирної їжі, схуднення. Лабораторні дослідження показали наявність стеатореї; кал гіпохолічний. Що може бути причиною такого стану? The patient notes frequent diarrhea, especially after eating fatty food, weight loss. Laboratory tests showed the presence of steatorrhea; stool is hypocholic. What could be the cause of this condition?

Незбалансована дієта Unbalanced diet

Обтурація жовчних шляхів Biliary obstruction

Недостатність панкреатичної ліпази Pancreatic lipase deficiency

Запалення слизової оболонки тонкої кишки Inflammation of the mucous membrane of the small intestine

Недостатність панкреатичної фосфо-ліпази Pancreatic phospholipase deficiency

63 / 200
Під час розтину тіла чоловіка 56 років, який страждав на фіброзно-кавернозний туберкульоз легень, знайдено збільшену у розмірах щільну селезінку. На розрізі тканина її коричнево-рожевого кольору, гладенька, з віскоподібною поверхнею. Який з перелічених патологічних процесів у селезінці найбільш вірогідний? During the autopsy of the body of a 56-year-old man who suffered from fibro-cavernous tuberculosis of the lungs, an enlarged, dense spleen was found. On the cut, the tissue is brownish-pink, smooth, with a waxy surface. Which of the listed pathological processes in the spleen is the most likely?

Цианотична індурація Cyanotic induration

Сальна селезінка Sebaceous Spleen

Сагова селезінка Sago spleen

Глазурна селезінка Glaze Spleen

Порфірна селезінка Porphyric spleen

64 / 200
У хворого з гнійничковими ураженнями шкіри виділений збудник, який на кров’яному агарі утворює жовті колонії округлої форми, середніх розмірів, оточені зоною гемолізу. У мазках з колоній - коки, розташовані скупченнями неправильної форми, грампозитивні. Виділена культура оксидазо- і каталазопозитивна, ферментує маніт, синтезує плазмокоагулазу. Який вид збудника виділений? In a patient with pustular lesions of the skin, the pathogen was isolated, which on blood agar forms yellow colonies of a rounded shape, of medium size, surrounded by a zone of hemolysis. In smears from colonies - cocci, located in clusters of irregular shape, are gram-positive. The isolated culture is oxidase- and catalase-positive, ferments mannitol, synthesizes plasma coagulase. What kind of pathogen is isolated?

Staphylococcus aureus Staphylococcus aureus

Staphylococcus epidermidis Staphylococcus epidermidis

Streptococcus agalactiae Streptococcus agalactiae

Streptococcus pyogenes Streptococcus pyogenes

Staphylococcus saprophyticus Staphylococcus saprophyticus

65 / 200
До стоматолога звернулася мати зі скаргами на руйнування зубів у дитини 2-х років. Під час огляду молочні зуби деформовані, уражені карієсом, у шийки коричнева облямівка. З анамнезу встановлено, що мати під час вагітності приймала антибіотики без контролю лікаря. Вкажіть, антибютики якої групи, що мають найбільш виражену тератогенну дію, могла приймати мати? A mother came to the dentist with complaints about the destruction of the teeth of a 2-year-old child. During the examination, the milk teeth are deformed, affected by caries, and there is a brown border around the neck. From the history it was established that the mother took antibiotics during pregnancy without a doctor's supervision. Specify which group of antibiotics with the most pronounced teratogenic effect could the mother have taken?

Цефалоспорини Cephalosporins

Аміноглікозиди Aminoglycosides

Пєніциліни Penicillins

Тетрацикліни Tetracyclines

Макроліди Macrolides

66 / 200
У хворого на хронічну серцеву недостатність у процесі лікування препаратами наперстянки виникли симптоми, що свідчать про початок токсичної дії серцевих глікозидів. Який препарат необхідно призначити для зменшення негативної дії серцевих глікозидів? During treatment with digitalis drugs, a patient with chronic heart failure developed symptoms indicating the onset of the toxic effect of cardiac glycosides. What drug should be prescribed to reduce the negative effect of cardiac glycosides ?

Діпіроксим Dipiroxime

Етімізол Etimizole

Атропіну сульфат Atropine sulfate

Кофеїн-бензоат натрію Caffeine sodium benzoate

Калію хлорид Potassium chloride

67 / 200
До лікаря звернувся хворий зі скаргами на непереносимість сонячної радіації. Мають місце опіки шкіри та порушення зору. Попередній діагноз: альбінізм. Порушення обміну якої амінокислоти відзначається у цього пацієнта? A patient came to the doctor with complaints of intolerance to solar radiation. There are skin burns and visual disturbances. Preliminary diagnosis: albinism. Which amino acid metabolism disorder is noted in this patient?

Тирозин Tyrosine

Лізин Lysine

Пролін Proline

Триптофан Tryptophan

Аланін Alanine

68 / 200
До лікарні швидкої допомоги доставили дитину 7 років у стані алергічного шоку, який розвинувся після того, як її вжалила оса. У крові підвищена концентрація гістаміну. В результаті якої реакції утворився цей амін? A 7-year-old child was brought to the emergency hospital in a state of allergic shock, which developed after being stung by a wasp. Histamine concentration in the blood is increased. As a result of which reaction was this amine formed?

Гідрооксилювання Hydroxylation

Відновлення Recovery

Декарбоксилювання Decarboxylation

Дегідрування Dehydrogenation

Дезамінування Demining

69 / 200
Під час огляду хворого лікар запідозрив синдром Іценка-Кушинга. Визначення якої речовини в крові хворого підтвердить припущення лікаря? During the patient's examination, the doctor suspected Itsenko-Cushing syndrome. The determination of which substance in the patient's blood will confirm the doctor's assumption?

Адреналін Adrenaline

Ретинол Retinol

Кортизол Cortisol

Токоферол Tocopherol

Холестерин Cholesterol

70 / 200
У дитини 6 місяців спостерігається різке відставання в психомоторному розвитку, напади судом, бліда шкіра з екзематозними змінами, біляве волосся, блакитні очі. У цієї дитини найбільш вірогідно дозволить встановити діагноз визначення концентрації у крові та сечі: A 6-month-old child has a sharp lag in psychomotor development, convulsions, pale skin with eczematous changes, blond hair, blue eyes. This child is most likely to be diagnosed diagnosis of determination of concentration in blood and urine:

Фенілпірувату Phenylpyruvate

Триптофану Tryptophan

Валінду Valindu

Гістидину Histidine

Лейцину Leucine

71 / 200
Тромбоз коронарної артерії спричинив розвиток інфаркту міокарда. Які механізми ушкодження кардіоміоцитів є домінуючими при цьому захворюванні? Thrombosis of the coronary artery caused the development of myocardial infarction. What mechanisms of damage to cardiomyocytes are dominant in this disease?

Кальцієві Calcium

Ацидотичні Acidotic

Протеїнові Protein

Електролітно-осмотичні Electrolyte-osmotic

Ліпідні Lipids

72 / 200
У крові хворого виявлено низький рівень альбумінів та фібриногену. Зниження активності яких органел гепатоцитів найбільш вірогідно обумовлює це явище? A low level of albumin and fibrinogen was detected in the patient's blood. A decrease in the activity of which organelle of hepatocytes most likely causes this phenomenon?

Лізосоми Lysosomes

Гранулярна ендоплазматична сітка Granular endoplasmic reticulum

Комплекс Гольджі Golgi Complex

Мітохондрії Mitochondria

Агранулярна ендоплазматична сітка Agranular endoplasmic reticulum

73 / 200
Під час обстеження хворого була виявлена недостатня кількість імуноглобулінів. Які з клітин імунної системи їх продукують? During the examination of the patient, an insufficient amount of immunoglobulins was detected. Which cells of the immune system produce them?

Т-супресори T-suppressors

Т-хелпери T-helpers

Т-кілери T-killers

Плазматичні Plasma

Плазмобласти Plasmoblasts

74 / 200
Жінка 33 років страждає на гепатоцеребральну дистрофію (хвороба Вільсона). У крові - знижений вміст церулоплазміну. У сечі - різко підвищений вміст амінокислот. Посиленням якого процесу зумовлені ці зміни? A 33-year-old woman suffers from hepatocerebral dystrophy (Wilson's disease). In the blood, the content of ceruloplasmin is reduced. In the urine, the content of amino acids is sharply increased. The strengthening of which process caused these changes ?

Ілюконеогенез Iluconeogenesis

Переамінування амінокислот Reamination of amino acids

Синтез сечовини Urea synthesis

Комплексоутворення амінокислот з міддю Complex formation of amino acids with copper

Розпад тканинних 6ілків Decay of fabric fibers

75 / 200
В одному з гірських селищ спостерігалася масова загибель гризунів, що супроводжувалася захворюванням мешканців селища. Для цієї хвороби було притаманне швидке підвищення t0 до 400 С, виражена інтоксикація, збільшення пахвинних лімфовузлів. У препаратах-мазках з трупного матеріалу виявлені грамнегативні палички овоїдної форми з біполярним забарвленням. Які мікроорганізми є збудниками цього інфекційного захворювання? In one of the mountain villages, a mass death of rodents was observed, which was accompanied by the disease of the inhabitants of the village. This disease was characterized by a rapid increase of t0 to 400 C, pronounced intoxication, an increase in inguinal lymph nodes. Gram-negative, ovoid-shaped rods with bipolar coloration were found in smear preparations from cadaver material. What microorganisms are the causative agents of this infectious disease?

Стафілокок Staphylococcus

Збудник туляремії The causative agent of tularemia

Збудник чуми Causative agent of the plague

Клостридії Clostridia

Збудник сибірки Causative agent of anthrax

76 / 200
До терапевтичного відділення надійшов хворий з тривалою бронхопневмонією. Антибіотикотерапія не дала належного ефекту. Який препарат для підвищення імунного статусу доцільно додати до комплексної терапії цього хворого? A patient with long-term bronchopneumonia was admitted to the therapeutic department. Antibiotic therapy did not have the proper effect. What drug to improve the immune status should be added to the complex therapy of this patient?

Тималін Tymalin

Аналгін Analgin

Димедрол Diphenhydramine

Парацетамол Paracetamol

Сульфокамфокаїн Sulfocamphocaine

77 / 200
У хворого на хронічний цистіт у біоптаті слизової оболонки сечового міхура разом з перехідним епітелієм виявлені вогнища багатошарового плоского незроговілого епітелію. Який процес лежить в основі зазначених змін в епітелії? Focuses of multi-layered flat non-keratinized epithelium were found in a biopsy of the mucous membrane of the urinary bladder in a patient with chronic cystitis along with transitional epithelium. What process is the basis of these changes in the epithelium?

Гіперкератоз Hyperkeratosis

Гіперплазія Hyperplasia

Метаплазія Metaplasia

Дисплазія Dysplasia

Дистрофія Dystrophy

78 / 200
Який механізм тепловіддачі найбільш ефективно реалізується під час перебування людини при температурі навколишнього середовища +350 С та 80% вологості повітря? Which heat transfer mechanism is most effectively implemented during a person's stay at an ambient temperature of +350 C and 80% air humidity?

Радіація Radiation

- -

Теплопроведення Heat conduction

Конвекція Convection

Випаровування Evaporation

79 / 200
У хворого спостерігається підвищення опору вигнанню крові з лівого шлуночка. При якому із перерахованих патологічних процесів може виникнути така ситуація? The patient has an increased resistance to the expulsion of blood from the left ventricle. In which of the listed pathological processes can this situation occur?

Аортальний стеноз Aortic stenosis

Емболія легеневої артерії Pulmonary embolism

Артеріальна гіпотензія Arterial hypotension

Мітральний стеноз Mitral stenosis

Недостатність аортального клапана Aortic valve insufficiency

80 / 200
У представників однієї з людських популяцій тіло подовжене, широка варіабельність зросту, знижений об’єм м’язової маси, подовжені кінцівки, зменшена у розмірах і об’ємі грудна клітка, підвищене потовиділення, знижені показники основного обміну та синтезу жирів. До якого адаптивного типу людей відноситься дана популяція? Representatives of one of the human populations have an elongated body, wide variability in height, reduced muscle mass, elongated limbs, reduced chest size and volume cage, increased sweating, decreased indicators of basic metabolism and synthesis of fats. What adaptive type of people does this population belong to?

Арктичний Arctic

Тип зони помірного клімату Temperate climate zone type

Гірський Mountain

Тропічний Tropical

Проміжний Intermediate

81 / 200
Під час гістологічного дослідження біопсійного матеріалу шлунка виявили малу кількість чи повну відсутність парієтальних клітин у залозах. Слизову оболонку якої ділянки шлунку вивчали? During the histological examination of the biopsy material of the stomach, a small number or complete absence of parietal cells in the glands was found. The mucous membrane of which part of the stomach was studied?

Пілоричний відділ Pyloric Department

Тіло Body

Дно Bottom

- -

Кардіальний відділ Cardiac Department

82 / 200
До травматологічного пункту доставлено хворого з ушкодженням нижньої кінцівки внаслідок прямого удару по внутрішній поверхні середньої третини гомілки. Перелом якого анатомічного утворення найбільш вірогідний? A patient with a lower extremity injury due to a direct impact on the inner surface of the middle third of the lower leg was brought to the trauma center. What anatomical structure is most likely fractured?

Діафіз великогомілкової кістки Tibia diaphysis

Дистальний епіфіз великогомілкової кістки Distal epiphysis of tibia

Проксимальний епіфіз малогомілкової кістки Proximal epiphysis of fibula

Проксимальний епіфіз великогомілкової кістки Proximal tibial epiphysis

Дистальний епіфіз малогомілкової кістки Distal epiphysis of fibula

83 / 200
Тестостерон та його аналоги збільшують масу скелетних м’язів, що дозволяє використовувати їх для лікування дистрофій. Взаємодією з яким клітинним субстратом зумовлена ця дія? Testosterone and its analogues increase the mass of skeletal muscles, which allows them to be used for the treatment of dystrophies. What cell substrate is this effect due to interaction with?

Рибосоми Ribosomes

Білки-активатори транскрипції Transcription activator proteins

Ядерні рецептори Nuclear receptors

Хроматин Chromatin

Мембранні рецептори Membrane receptors

84 / 200
У вогнищі запалення підвищується проникність судин мікроциркуляторного русла, у них збільшується гідродинамічний опір. У міжклітинній рідині підвищується осмотична концентрація та дисперсність білкових структур. Який вид набряку буде спостерігатися у даному випадку? In the focus of inflammation, the permeability of the vessels of the microcirculatory bed increases, their hydrodynamic resistance increases. In the intercellular fluid, the osmotic concentration and dispersion of protein structures increases. What type of edema will be observed in this case?

Колоїдно-осмотичний Colloid-osmotic

Змішаний Mixed

Гідродинамічний Hydrodynamic

Лімфогенний Lymphogenic

Мембраногенний Membranogenic

85 / 200
Під час дослідження коронарних артерій виявлені атеросклеротичні бляшки з кальцинозом, що закривають просвіт судин на 1/3. У м’язі дрібні множинні білуваті прошарки сполучної тканини. Як називається процес, виявлений у міокарді? During the study of the coronary arteries, atherosclerotic plaques with calcification were found, covering 1/3 of the lumen of the vessels. In the muscle, there are small multiple whitish layers of connective tissue. What is the name the process detected in the myocardium?

Міокардит Myocarditis

Післяінфарктний кардіосклероз Postinfarction cardiosclerosis

Інфаркт міокарда Myocardial infarction

Тигрове серце Tiger Heart

Дифузний кардіосклероз Diffuse cardiosclerosis

86 / 200
У процесі онтогенезу у людини на організменному рівні проявилися наступні зміни: зменшилася життєва ємність легень, збільшився артеріальний тиск, розвинувся атеросклероз. Який період онтогенезу найбільш вірогідний у цьому випадку? In the process of ontogenesis, the following changes appeared in humans at the organismal level: the vital capacity of the lungs decreased, blood pressure increased, atherosclerosis developed. What period of ontogenesis is most likely in this case?

Юнацький Youth

Похилий вік Old age

Молодий вік Young age

Початок зрілого віку Beginning of adulthood

Підлітковий Teenager

87 / 200
Хворий 50 років з метою лікування черевного тифу почав приймати левоміцетин, але на наступний день його стан погіршився, температура підвищилася до 39,60 С. Чим пояснити погіршення стану хворого? A 50-year-old patient started taking chloramphenicol to treat typhoid fever, but the next day his condition worsened, the temperature rose to 39.60 C. How to explain the deterioration of the patient's condition ?

Дією ендотоксинів збудника The action of the pathogen's endotoxins

Алергічною реакцією Allergic reaction

Нечутливістю збудника до левоміцетину Insensitivity of the pathogen to chloramphenicol

Реінфекцією Reinfection

Приєднанням вторинної інфекції Addition of secondary infection

88 / 200
Як зміниться фармакологічна активність препарату з високою спорідненістю до білків плазми крові при виникненні гіпоальбумінемії? How will the pharmacological activity of a drug with high affinity to blood plasma proteins change when hypoalbuminemia occurs?

Підвищиться Will increase

Зникне Disappear

Не зміниться Will not change

Суттєво зменшиться Will significantly decrease

Дещо зменшиться It will decrease a bit

89 / 200
У людини в стані спокою значно збільшена робота м’язів видиху. Що з наведеного може бути причиною цього? In a person at rest, the work of expiratory muscles is significantly increased. Which of the following can be the reason for this?

Рідке дихання Liquid Breath

Звуження дихальних шляхів Narrowing of airways

Зменшення хвилинного об’єму дихання Decrease in minute respiratory volume

Поверхневе дихання Surface breathing

Негативний внутрішньоплевральний тиск Negative intrapleural pressure

90 / 200
У місті епідемія грипу. Який препарат доцільно використати для неспецифічної профілактики захворювання? There is an epidemic of influenza in the city. What drug should be used for non-specific prevention of the disease?

Протигрипозний імуноглобулін Anti-influenza immunoglobulin

Протигрипозна сироватка Anti-influenza serum

Протигрипозна вакцина Influenza vaccine

Пеніцилін Penicillin

Лейкоцитарний інтерферон Leukocyte interferon

91 / 200
Під час розтину трупа чоловіка 56 років у термінальному відділі тонкої кишки виявлено декілька виразок діаметром 4-5 см. Краї виразок піднімаються над поверхнею слизової оболонки; стінки вкриті сірувато-жовтуватими масами, які кришаться. Реакція Відаля позитивна. Який найбільш вірогідний діагноз? During the autopsy of the corpse of a 56-year-old man, several ulcers with a diameter of 4-5 cm were found in the terminal part of the small intestine. The edges of the ulcers rise above the surface of the mucous membrane; the walls are covered with grayish yellowish masses that crumble. Vidal's reaction is positive. What is the most likely diagnosis?

Дизентерія Dysentery

Поворотний тиф Typhoid

Хвороба Крона Crohn's disease

Черевний тиф Typhoid

Паратиф Paratif

92 / 200
Під час огляду дитини, яка перехворіла на кір, у м’яких тканинах щік та промежини виявлено нечітко відмежовані, набряклі, червоно-чорного кольору ділянки, у яких виявляється помірна флюктуація. Яке ускладнення розвинулося у дитини? During the examination of a child with measles, indistinctly demarcated, swollen, red-black areas were found in the soft tissues of the cheeks and perineum, in which moderate fluctuation. What complication did the child develop?

Трофічна виразка Tropical ulcer

Газова гангрена Gas gangrene

Пролежень Decubitus bedsores

Суха гангрена Dry gangrene

Волога гангрена (нома) Wet gangrene (noma)

93 / 200
У людини вимірють внутрішньо-плевральний тиск. У якій фазі людина затримала дихання, якщо величина тиску дорівнює - 7,5 см вод.ст.? A person's intra-pleural pressure will be measured. In what phase did the person hold his breath, if the pressure value is - 7.5 cm H2O?

Форсований видих Forced expiration

- -

Спокійний вдих Calm Breath

Форсований вдих Forced inhalation

Спокійний видих Calm Exhalation

94 / 200
В умовах експерименту у кролика перев’язали ниркову артерію, що через 2 тижні призвело до суттєвого збільшення артеріального тиску. У результаті збільшення секреції якої біологічно активної речовини це відбулося? Under experimental conditions, a rabbit's renal artery was ligated, which after 2 weeks led to a significant increase in blood pressure. As a result of increased secretion of which biologically active substance, this occurred?

Норадреналін Noradrenaline

Адреналін Adrenaline

Натрійуретичний гормон Natriuretic hormone

Ренін Renin

Вазопресин Vasopressin

95 / 200
Куди треба провести катетер для забору лімфи з грудної лімфатичної протоки? Where should the catheter be inserted to collect lymph from the thoracic lymphatic duct?

У нижню порожнисту вену Into the inferior vena cava

У ліву пахвинну вену To the left inguinal vein

У лівий венозний кут To the left venous corner

У правий венозний кут To the right venous corner

У верхню порожнисту вену Into the superior vena cava

96 / 200
У дитячому садку планується проведення вакцинації проти коклюшу. Який препарат необхідно використати з цією метою? In kindergarten, vaccination against whooping cough is planned. What drug should be used for this purpose?

Типоспецифічна сироватка Type-specific serum

Вакцина АКДП AKDP vaccine

Вакцина БЦЖ BCG vaccine

Нормальний у-глобулін Normal y-globulin

АДП анатоксин ADP toxoid

97 / 200
У студента перед іспитом виникла тахікардія. Які зміни на ЕКГ свідчитимуть про її наявність? The student developed tachycardia before the exam. What changes on the ECG would indicate its presence?

Подовження комплексу QRS Prolongation of the QRS complex

Подовження сегменту Q-T Prolongation of the QT segment

Подовження інтервалу R-R RR interval extension

Вкорочення інтервалу P-Q Shortening PQ interval

Вкорочення інтервалу R-R RR interval shortening

98 / 200
Перед відрядженням за кордон лікарю, з метою профілактики малярії, призначено засіб з гістомизонтоцидною дією. Який препарат він одержував? Before traveling abroad, the doctor was prescribed a histomyzontocidal drug to prevent malaria. What drug did he receive?

Бісептол Biseptol

Доксициклін Doxycycline

Хінін Quinine

Примахін Primakhin

Хлоридин Chloridine

99 / 200
У хлопчика 5 років, якого непокоїть анальний свербіж, виявлені черв’яки класу нематод (гострики). Оберіть лікарський засіб для дегельмінтації дитини: A 5-year-old boy who is worried about anal itching, worms of the nematode class (pinworms) were found. Choose a medicine for deworming the child:

Сім’я гарбуза Pumpkin Family

Аміноакрихін Aminoacrychin

Фенасал Fenasal

Мебендазол Mebendazole

Празіквантель Praziquantel

100 / 200
Після ремонту автомобілю в гаражному приміщенні водій потрапив до лікарні з симптомами отруєння вихлопними газами. Вміст якої речовини у крові буде підвищено? After repairing the car in the garage, the driver was hospitalized with symptoms of exhaust gas poisoning. What substance will be elevated in the blood?

Метгемоглобін Methemoglobin

Карбоксигемоглобін Carboxyhemoglobin

Карбгемоглобін Carbhemoglobin

Глікозильований гемоглобін Glycosylated hemoglobin

Оксигемоглобін Oxyhemoglobin

101 / 200
У жінки через 6 місяців після пологів розвинулася маткова кровотеча. Під час гінекологічного обстеження у порожнині матки виявлена тканина темно-червоного кольору з множинними порожнинами, що нагадує 'губку'. Під час мікроскопічного дослідження пухлини у лакунах крові виявлені атипові світлі епітеліальні клітини Лангханса та гігантські клітини синцитіотрофобласта. Яка це пухлина? A woman developed uterine bleeding 6 months after giving birth. During a gynecological examination, a dark red tissue with multiple cavities resembling a 'sponge' was found in the uterine cavity During the microscopic examination of the tumor, atypical light Langhans epithelial cells and giant syncytiotrophoblast cells were found in the blood lacunae. What kind of tumor is this?

Фіброміома Fibroid

Хоріонепітеліома Chorioepithelioma

Міхуровий занос Bubble drift

Аденокарцинома Adenocarcinoma

Плоскоклітинний незроговілий рак Squamous non-keratinous carcinoma

102 / 200
Хвора 13 років знаходиться на стаціонарному лікуванні в гематологічному відділенні обласної дитячої лікарні з діагнозом залізодефіцитна анемія. Який тип гіпоксії має місце у цієї хворої? A 13-year-old patient is undergoing inpatient treatment in the hematology department of the regional children's hospital with a diagnosis of iron deficiency anemia. What type of hypoxia does this patient have?

Тканинна Fabric

Змішана Mixed

Дихальна Respiratory

Циркуляторна Circulator

Гемічна Chemical

103 / 200
Хворому, що страждає на тромбоемболічну хворобу, призначений штучний антикоагулянт пелентан. Антагоністом якого вітаміну є цей препарат? A patient suffering from a thromboembolic disease was prescribed the artificial anticoagulant Pelentan. Which vitamin is an antagonist of this drug?

Вітамін Е Vitamin E

Вітамін D Vitamin D

Вітамін А Vitamin A

Вітамін К Vitamin K

Вітамін С Vitamin C

104 / 200
У хворої гінекологічного відділення виникли симптоми внутрішньої кровотечі. Який засіб слід призначити з метою пригнічення фібринолізу та зупинки кровотечі? A patient in the gynecological department developed symptoms of internal bleeding. What drug should be prescribed to suppress fibrinolysis and stop the bleeding?

Дицинон Dizinon

Фібриноген Fibrinogen

Вікасол Vikasol

Контрикал Contrical

Хлористий кальцій Calcium Chloride

105 / 200
У хворого з клінічними ознаками первинного імунодефіциту виявлено порушення функції антигенпрезентації імунокомпетентним клітинам. Порушення функціонування яких клітін може бути причиною цього? In a patient with clinical signs of primary immunodeficiency, a violation of the function of antigen presentation to immunocompetent cells was detected. A violation of the functioning of which cells can be the cause of this?

Т-лімфоцити T-lymphocytes

В-лімфоцити B-lymphocytes

Макрофаги, моноцити Macrophages, monocytes

Фібробласти Fibroblasts

0-лімфоцити 0-lymphocytes

106 / 200
У хворого 37 років на фоні тривалого застосування антибіотиків спостерігається підвищена кровоточивість при невеликих пошкодженнях. У крові - зниження активності II, VII, X факторів згортання крові; подовження часу згортання крові. Нестачею якого вітаміну обумовлені ці зміни? In a 37-year-old patient, against the background of long-term use of antibiotics, there is increased bleeding with small injuries. In the blood, there is a decrease in the activity of II, VII, X blood coagulation factors; prolongation of coagulation time blood. The lack of which vitamin causes these changes?

Вітамін А Vitamin A

Вітамін С Vitamin C

Вітамін D Vitamin D

Вітамін К Vitamin K

Вітамін Е Vitamin E

107 / 200
У хворого на хронічний гломерулонефрит швидкість клубочкової фільтрації (ШКФ) знижена до 20% від нормальної. Що є основною причиною зниження ШКФ у даному випадку? In a patient with chronic glomerulonephritis, the glomerular filtration rate (GFR) is reduced to 20% of normal. What is the main reason for the decrease in GFR in this case?

Тромбоз ниркових артерій Thrombosis of renal arteries

Тубулопатія Tubulopathy

Обтурація сечовивідних шляхів Obstruction of the urinary tract

Зменшення кількості функціонуючих нефронів Decreasing the number of functioning nephrons

Ішемія нирок Kidney ischemia

108 / 200
5 років температура підвищилася до 40о 108. С, з’явився різкий головний біль, блювання, неспокій, озноб. Через 4 дні з’явилася геморагічна висипка на шкірі, олігоурія та надниркова недостаність, що і стало причиною смерті. При бактеріологічному дослідженні мазків з глотки виявлено менінгокок. Яка форма хвороби виявлена? 5 years ago, the temperature rose to 40° 108. C, a sharp headache, vomiting, restlessness, chills appeared. After 4 days, a hemorrhagic rash appeared on the skin , oliguria and adrenal insufficiency, which was the cause of death. Bacteriological examination of swabs from the pharynx revealed meningococcus. What form of the disease was detected?

Менінгококовий менінгіт Meningococcal meningitis

Менінгококцемія Meningococcemia

Менінгоенцефаліт Meningoencephalitis

Гідроцефалія Hydrocephalus

Менінгококовий назофарингіт Meningococcal nasopharyngitis

109 / 200
У жінки 37 років під час оперативного втручання на органах малого тазу виникла необхідність перев’язати маткову артерію. Яке з утворень може бути випадково перев’язаним разом з нею? A 37-year-old woman had to ligate the uterine artery during surgery on the pelvic organs. Which of the formations can be accidentally ligated together with it?'

Кругла зв’язка матки Round ligament of the uterus

Внутрішня клубова вена Internal iliac vein

Сечовід Ureter

Сечівник Urine

Маткова труба Uterine tube

110 / 200
Під час гістологічного дослідження тимуса чоловіка 40 років, визначено зменшення частки паренхіматозних елементів залози, збільшення частки жирової та пухкої сполучної тканини, збагачення її тимусними тільцями при незмінній загальній масі органу. Як зветься таке явище? During histological examination of the thymus of a 40-year-old man, a decrease in the proportion of parenchymal elements of the gland, an increase in the proportion of adipose and loose connective tissue, and its enrichment with thymus bodies were determined while the total mass of the organ remained unchanged What is this phenomenon called?

Акцідентальна інволюція Accidental Involution

Дистрофія Dystrophy

Атрофія Atrophy

Гіпотрофія Hypotrophy

Вікова інволюція Age involution

111 / 200
Підліток 12 років протягом 3 місяців втратив 7 кг маси тіла. Вміст глюкози у крові становить 20 ммоль/л. Несподівано розвинулася кома. Який вид цукрового діабету найбільш вірогідний у хлопчика? A 12-year-old boy lost 7 kg of body weight in 3 months. Blood glucose is 20 mmol/l. A coma suddenly developed. What type of diabetes is most likely in a boy?

Стероїдний Steroid

Гіпертіреоїдний Hyperthyroid

Гіпофізарний Pituitary

Інсулінозалежний (I тип) Insulin dependent (I type)

Інсулінонезалежний (II тип) Insulin independent (II type)

112 / 200
Внаслідок тривалого голодування в організмі людини швидко зникають резерви вуглеводів. Який з процесів метаболізму підтримує при цьому вміст глюкози в крові? As a result of prolonged starvation in the human body, reserves of carbohydrates quickly disappear. Which of the metabolic processes supports the glucose content in the blood?

Анаеробний гліколіз Анаеробний гліколіз

Пентозофосфатний цикл Пентозофосфатний цикл

Аеробний гліколіз Аеробний гліколіз

Глікогеноліз Глікогеноліз

Глюконеогенез Gluconeogenesis

113 / 200
У чоловіка 55 років, який протягом багатьох років страждає на недостатність мітрального клапана, виникла гостра серцева недостатність. Який патофізіологічний варіант недостатності серця спостерігається у цьому випадку? У чоловіка 55 років, який протягом багатьох років страждає на недостатність мітрального клапана, виникла гостра серцева недостатність. Який патофізіологічний варіант недостатності серця спостерігається у цьому випадку?

Гостра тампонада серця Acute cardiac tamponade

Коронарогенне ушкодження серця Коронарогенне ушкодження серця

Перевантаження серця об’ємом Перевантаження серця об’ємом

Гіпоксичне ушкодження серця Гіпоксичне ушкодження серця

Нейрогенне ушкодження серця Нейрогенне ушкодження серця

114 / 200
У бактеріологічній лабораторії проводиться дослідження м‘ясних консервів на вміст ботулінічного токсину. Для цього дослідній групі мишей ввели екстракт із досліджуваного матеріалу та антитоксичну протиботулінічну сироватку типів А, В, Е; контрольній групі мишей ввели екстракт без протиботулінічної сироватки. Яку серологічну реакцію було використано? In the bacteriological laboratory, canned meat is being tested for the content of botulinum toxin. For this, the experimental group of mice was injected with an extract from the researched material and antitoxic antibotulinum serum of types A, B, E; a control group of mice was injected with the extract without anti-botulinum serum. What serological reaction was used?

Опсоно-фагоцитарна Opsono-phagocytic

Подвійної імунної дифузії Double immune diffusion

Преципітації Precipitation

Зв‘язування комплементу Complement Binding

Нейтралізації Neutralizations

115 / 200
Хворому, що страждає на хронічну серцеву недостатність, лікар порадив провести профілактичний курс лікування кардіотонічним препаратом з групи серцевих глікозидів, який приймають ентерально. Який препарат було рекомендовано хворому? A patient suffering from chronic heart failure was advised by the doctor to conduct a preventive course of treatment with a cardiotonic drug from the group of cardiac glycosides, which is taken enterally. What drug was recommended to the patient?

Корглікон Corglycon

Кордіамін Cordiamine

Дигоксин Digoxin

Строфантин Strophantin

Кордарон Cordaron

116 / 200
Офтальмолог з діагностичною метою (розширення зіниць для огляду очного дна) використав 1 % розчин мезатону. Чим обумовлений мідріаз, викликаний препаратом? An ophthalmologist used a 1% mesaton solution for diagnostic purposes (pupil dilation to examine the fundus). What causes mydriasis caused by the drug?

Активація β1 адренорецепторів Activation of β1 adrenoceptors

Активація α2 адренорецепторів Activation of α2 adrenoceptors

Активація М-холінорецепторів Activation of M-cholinergic receptors

Активація α1 адренорецепторів Activation of α1 adrenoceptors

Блокада α1 адренорецепторів Blockade of α1 adrenoceptors

117 / 200
Чоловіка 45 років протягом останніх 3 років непокоїв сухий кашель; наростала задишка, легенева недостатність, швидко втрачалася вага. На розтині: легеневе серце; у легенях різко виражений фіброз з наявністю порожнин, що створюють картину 'медових сот'. Гістологічно: інтерстиційний фіброз з вираженою інфільтрацією строми лімфогістіоцитами з домішками нейтрофілів. Який найбільш вірогідний діагноз? A 45-year-old man has been troubled by a dry cough for the past 3 years; shortness of breath, pulmonary insufficiency, and rapid weight loss. At autopsy: pulmonary heart; in the lungs, marked fibrosis with by the presence of cavities creating a 'honeycomb' picture. Histologically: interstitial fibrosis with marked infiltration of the stroma by lymphohistiocytes with admixtures of neutrophils. What is the most likely diagnosis?

Фіброзуючий альвеоліт Fibrosing alveolitis

Хронічна бульозна емфізема Chronic bullous emphysema

Післязапальний пневмосклероз Post-inflammatory pneumosclerosis

Бронхоектатична хвороба Bronchoectatic disease

Пиловий пневмосклероз Dust pneumosclerosis

118 / 200
Під час огляду дитини педіатр відзначив відставання у фізичному та розумовому розвитку. У сечі різко підвищений вміст кетокислоти, що дає якісну кольорову реакцію з хлорним залізом. Яке порушення обміну речовин було виявлено? During the examination of the child, the pediatrician noted a delay in physical and mental development. The urine has a sharply increased content of ketoacid, which gives a high-quality color reaction with iron chloride. What a metabolic disorder it was found?

Тирозинемія Tyrosinemia

Фенілкетонурія Phenylketonuria

Альбінізм Albinism

Алкаптонурія Alkaptonuria

Цистинурія Cystinuria

119 / 200
Хвора 40 років надійшла до інфекційного відділення лікарні з високою температурою тіла. Об’єктивно: виражені менінгеальні симптоми. Проведено спиномозкову пункцію. Яке анатомічне утворення було пропунктовано? A 40-year-old patient was admitted to the hospital's infectious disease department with a high body temperature. Objectively: pronounced meningeal symptoms. A spinal tap was performed. What anatomical formation was punctured?

Spatium subarachnoideum Spatium subarachnoideum

Spatium subdurale Spatium subdurale

Cisterna cerebellomedullaris posterior Cisterna cerebellomedullaris posterior

Cavum trigeminale Cavum trigeminale

Spatium epidurale Spatium epidurale

120 / 200
Тварині внутрішньовенно ввели концентрований розчин хлориду натрію, що зумовило зниження реабсорбції іонів натрію у канальцях нирок. Внаслідок яких змін секреції гормонів це відбувається? The animals were intravenously injected with a concentrated solution of sodium chloride, which led to a decrease in the reabsorption of sodium ions in the tubules of the kidneys. What changes in the secretion of hormones does this occur?

Збільшення вазопресину Vasopressin Increase

Зменшення альдостерону Aldosterone reduction

Зменшення натрійуретичного фактора Reduction of natriuretic factor

Зменшення вазопресину Vasopressin reduction

Збільшення альдостерону Aldosterone Increase

121 / 200
У холодну погоду з вітром люди замерзають швидше, ніж за відсутності вітру. Причиною цього є те, що вітер, вперш за все, збільшує віддачу тепла шляхом: In cold weather with wind, people freeze more quickly than in the absence of wind. The reason for this is that the wind increases heat transfer primarily by:

Радіації Radiation

Випаровування Evaporation

- -

Конвекції Convections

Теплопроведення Heat conduction

122 / 200
Під час бігу на довгі дистанції скелетна мускулатура тренованої людини використовує глюкозу з метою отримання енергії АТФ для м’язового скорочення. Вкажіть основний процес утилізації глюкози в цих умовах: During long-distance running, the skeletal muscle of a trained person uses glucose to obtain ATP energy for muscle contraction. State the main process of glucose utilization under these conditions:

Ілікогенез Ilicogenesis

Аеробний гліколіз Aerobic glycolysis

Ілюконеогенез Iluconeogenesis

Анаеробний гліколіз Anaerobic glycolysis

Ілікогеноліз Elicogenolysis

123 / 200
Дитина 10 років під час гри порізала ногу відламком скла та була направлена до поліклініки для введення протиправцевої сироватки. З метою попередження розвитку анафілактичного шоку лікувальну сироватку вводили за Безредкою. Який механізм лежить в основі подібного способу гіпосенсибілізації організму? A 10-year-old child cut her leg with a shard of glass while playing and was sent to the polyclinic for administration of anti-tetanus serum. In order to prevent the development of anaphylactic shock, the therapeutic serum was administered according to Bezredka. Which the mechanism underlies this method of hyposensitization of the body?

Зв’язування рецепторів до IgE на тучних клітинах IgE receptor binding on mast cells

Зв’язування фіксованих на тучних клітинах IgE Mat cell-fixed IgE binding

Стимуляція імунологічної толерантності до антигену Stimulation of immunological tolerance to antigen

Стимуляція синтезу антиген-специфічних IgG2 Stimulation of the synthesis of antigen-specific IgG2

Блокування синтезу медіаторів тучних клітин Blocking the synthesis of mast cell mediators

124 / 200
У хворого через добу після апендектомії у крові визначається нейтрофільний лейкоцитоз із регенеративним зсувом. Який найбільш вірогідний механізм розвитку лейкоцитозу у даному випадку? One day after appendectomy, neutrophilic leukocytosis with a regenerative shift is detected in the patient's blood. What is the most likely mechanism of development of leukocytosis in this case?

Посилення лейкопоезу Increasing leukopoiesis

Уповільнення руйнування лейкоцитів Slowing down the destruction of leukocytes

Посилення лейкопоезу та уповільнення еміграції лейкоцитів у тканини Strengthening of leukopoiesis and slowing down of emigration of leukocytes into tissues

Перерозподіл лейкоцитів у організмі Redistribution of leukocytes in the body

Уповільнення еміграції лейкоцитів у тканини Slowing down of emigration of leukocytes into tissues

125 / 200
У хворого з широким інфарктом міокарда розвинулася серцева недостатність. Який патогенетичний механізм її розвитку? A patient with a wide myocardial infarction developed heart failure. What is the pathogenetic mechanism of its development?

Перевантаження серця тиском Heart pressure overload

Зменшення маси функціонуючих кардіоміоцитів Decreasing the mass of functioning cardiomyocytes

Перевантаження серця об’ємом Heart volume overload

Реперфузійне ураження міокарда Myocardial reperfusion injury

Гостра тампонада серця Acute cardiac tamponade

126 / 200
Під час визначення групової належності крові за системою АВ0, аглютинацію еритроцитів досліджуваної крові викликали стандартні сироватки I та II груп, та не викликала сироватка III групи. Кров якої групи досліджується? During the blood group determination according to the AB0 system, standard sera of groups I and II caused agglutination of erythrocytes of the examined blood, but not group III serum. Which blood group is being examined ?

А(II)β А(II)β

Неможливо визначити Unable to determine

В(III)α B(III)α

АВ(IV) AB(IV)

O(I)α та β O(I)α and β

127 / 200
Хворий 50 років скаржиться на спрагу, п’є багато води; виражена поліурія. Глюкоза крові - 4,8 ммоль/л. У сечі глюкози та ацетонових тіл немає, сеча безбарвна, питома вага- 1,002 - 1,004. Яка причина поліурії? A 50-year-old patient complains of thirst, drinks a lot of water; pronounced polyuria. Blood glucose - 4.8 mmol/l. There is no glucose or acetone bodies in the urine , urine is colorless, specific gravity - 1.002 - 1.004. What is the cause of polyuria?

Інсулінова недостатність Insulin deficiency

Альдостеронізм Aldosteronism

Гіпотиреоз Hypothyroidism

Нестача вазопресину Нестача вазопресину

Тиреотоксикоз Thyrotoxicosis

128 / 200
До ендокринолога звернулася хвора 45 років із скаргами на підвищенний апетит, сухість слизових оболонок ротової порожнини, зростання діурезу. При обстеженні вперше виявлено інсуліннезалежний діабет. Який з названих препаратів доцільно призначити хворій? До ендокринолога звернулася хвора 45 років із скаргами на підвищенний апетит, сухість слизових оболонок ротової порожнини, зростання діурезу. При обстеженні вперше виявлено інсуліннезалежний діабет. Який з названих препаратів доцільно призначити хворій?

Інсулін Інсулін

Глібенкламід Глібенкламід

Вазопресин Вазопресин

Окситоцин Окситоцин

Адіурекрин Адіурекрин

129 / 200
До реанімаційного відділення надійшов хворий із симптомами гострого отруєння морфіном - непритомність, гіпотермія, дихання Чейн-Стокса, гіпотензія, брадикардія, міоз. Який з перерахованих препаратів буде найефективнішим у даній ситуації? A patient was admitted to the intensive care unit with symptoms of acute morphine poisoning - fainting, hypothermia, Cheyne-Stokes breathing, hypotension, bradycardia, miosis. Which of the listed drugs will be the most effective in this situation?

Кордіамін Cordiamine

Кофеїн Caffeine

Налоксон Naloxone

Етимізол Etimizole

Камфора Camphor

130 / 200
До лікаря звернувся хворий із скаргами на біль у лівій половині шиї, який посилюється при рухах голови. Положення при якому біль не турбує - це нахил голови ліворуч з поверненим обличчям праворуч. Ураження якого м’яза спричиняє біль у даному випадку? A patient came to the doctor with complaints of pain in the left half of the neck, which worsens with head movements. The position in which the pain does not bother is tilting the head to the left with the face turned on the right. What muscle damage causes pain in this case?

M.stemocleidomastoideus dexter M. stemocleidomastoideus dexter

M.platisma sister M.platisma sister

М.stemocleidomastoideu sinister M. stemocleidomastoideu sinister

M.platisma dexter M.platisma dexter

M.longus со11і M.longus со11и

131 / 200
Хвора у віці 69 років страждає хронічними запорами, в основі яких лежить гіпотонія товстої кишки. Який з перерахованих препаратів доцільно призначити? A 69-year-old patient suffers from chronic constipation, the basis of which is colonic hypotonia. Which of the listed drugs should be prescribed?

Натрію сульфат Sodium sulfate

Прозерин Prozerin

Бісакоділ Bisacodil

Магнію сульфат Magnesium sulfate

Касторова олія Castor oil

132 / 200
Хворий 45 років госпіталізований до лікарні зі скаргами на високу температуру, біль під час дихання, задишку та кашель. Після обстеження, лабораторної та рентгенодіагностики було встановлено діагноз - плеврит. Для евакуації ексудату була призначена плевральна пункція. В якому місці плевральної порожнини буде знаходитися найбільша кількість ексудату? A 45-year-old patient was hospitalized with complaints of high temperature, pain during breathing, shortness of breath and cough. After examination, laboratory and x-ray diagnostics, the diagnosis of pleurisy was established. A pleural puncture was prescribed to evacuate the exudate. Where in the pleural cavity will the largest amount of exudate be located?

Під коренем легенів Under the root of the lungs

Реберно-медіастинальний синус Cost-mediastinal sinus

Діафрагмально-медіастинальний синус Diaphragmatic-mediastinal sinus

Під куполом плеври Under the dome of the pleura

Реберно-діафрагмальний синус Cost-phrenic sinus

133 / 200
Під час розтину трупа жінки 69 років підвищеного живлення, яка померла від гострого інфаркту міокарда, в інтимі коронарних артерій знайдені численні білуваті, щільні, вибухаючі формування, що різко звужують просвіт судин. Для якої стадії атеросклерозу властиві такі зміни? During the autopsy of a 69-year-old well-nourished woman who died of an acute myocardial infarction, numerous whitish, dense, exploding formations were found in the intima of the coronary arteries, sharply narrowing vascular lumen. For which stage of atherosclerosis are these changes typical?

Атерокальциноз Atherocalcinosis

Ліпосклероз Liposclerosis

Атероматоз Atheromatosis

Стадія утворення атероматозної виразки Stage of formation of atheromatous ulcer

Ліпоїдоз Lipoidosis

134 / 200
Під час обстеження чоловіка 45 років, який перебуває довгий час на вегетеріанській рослинній дієті, виявлено негативний азотистий баланс. Яка особливість раціону стала причиною цього? During the examination of a 45-year-old man who has been on a vegetarian plant-based diet for a long time, a negative nitrogen balance was found. What feature of the diet caused this?

Недостатня кількість білків Not enough proteins

Недостатня кількість вітамінів Insufficient amount of vitamins

Недостатня кількість жирів Not enough fats

Надлишкова кількість вуглеводів Excess amount of carbohydrates

Надлишкова кількість води

135 / 200
Хворій на ревматоїдний поліартрит призначили нестероїдний протизапальний засіб - диклофенак натрію. Через деякий час у хворої виникло загострення супутнього захворювання, що змусило відмінити препарат. Яке супутнє захворювання могло призвести до відміни препарату? A patient with rheumatoid polyarthritis was prescribed a non-steroidal anti-inflammatory drug - diclofenac sodium. After some time, the patient developed an exacerbation of a concomitant disease, which forced the drug to be discontinued. What concomitant disease could lead to cancel the drug?

Гіпертонічна хвороба Hypertensive disease

Цукровий діабет Diabetes

Виразкова хвороба шлунка Gastric ulcer disease

Бронхіальна астма Bronchial asthma

Ішемічна хвороба серця Ischemic heart disease

136 / 200
Хворий, що лікувався з приводу неврозу сибазоном, скаржиться на зубний біль. Лікар призначив йому знеболювальний засіб у дозі, яка меньша за середню терапевтичну. Яке явище взяв до уваги лікар, зменшуючи дозу препарату? A patient treated for neurosis with sibazon complains of a toothache. The doctor prescribed him a painkiller in a dose that is lower than the average therapeutic dose. What phenomenon did you take into account doctor, reducing the dose of the drug?

Толерантність Tolerance

Сумація Summation

Потенціювання Potentiation

Лікарська залежiсть Drug addiction

Кумуляція Cumulative

137 / 200
Під час дослідження сироватки крові пацієнта з ознаками імунодефіциту виявлено антитіла до білків gP120 та gP41. Наявність якої інфекції у хворого підтверджує цей результат? During the examination of the blood serum of a patient with signs of immunodeficiency, antibodies to gP120 and gP41 proteins were detected. The presence of which infection in the patient confirms this result?

ВІЛ-інфекція HIV infection

ЕСНО-інфекція ESNO infection

HBV-інфекція HBV infection

HLTV-1-інфекція HLTV-1 infection

TORCH-інфекція TORCH infection

138 / 200
На розтині тіла чоловіка 46 років на слизовій оболонці прямої та сигмовидної кишок виявилено множинні коричнево-зелені нашарування, крововиливи; у просвіті кишки слиз, невелика кількість крові; гістологічно - фібринозний коліт. При бактеріологічному дослідженні вмісту кишки висіяно S.Sonne. Який найбільш вірогідний діагноз? At the autopsy of the body of a 46-year-old man, multiple brown-green layers, hemorrhages were found on the mucous membrane of the rectum and sigmoid colon; in the lumen of the intestine, mucus, a small amount of blood; histologically - fibrinous colitis. Bacteriological examination of the intestinal contents revealed S.Sonne. What is the most likely diagnosis?

Хвороба Крона Crohn's disease

Сальмонельоз Salmonellosis

Холера Cholera

Ієрсініоз Yersiniosis

Дизентерія Dysentery

139 / 200
Під час розтину тіла чоловіка, який служив на підводному атомному човні, виявили наступне: спустошення в кістковому мозку (панмієлофтиз), анемія, лейкопенія, тромбоцитопенія, розпад лімфоцитів у лімфатичних вузлах, селезінці, лімфатичному апараті шлунково-кишкового тракту, крововиливи в на-днирники. Який найбільш вірогідний діагноз? During the autopsy of the body of a man who served on a nuclear submarine, the following was found: devastation in the bone marrow (panmyelophthisis), anemia, leukopenia, thrombocytopenia, disintegration of lymphocytes in lymph nodes, spleen, lymphatic apparatus of the gastrointestinal tract, hemorrhages in the adrenal glands. What is the most likely diagnosis?

Кесонна хвороба Casson disease

Гострий лейкоз Acute leukemia

Гостра променева хвороба Acute radiation sickness

Гостра анемія Acute anemia

Вібраційна хвороба Vibration disease

140 / 200
Пацієнтка 58 років скаржиться на підвищену втомлюваність, зниження працездатності, сонливість, задишку під час швидкої ходи. У крові: ер.-4,61012/л, Hb- 92 г/л, кол.показн.- 0,6. У мазку

крові велика кількість анулоцитів та мікроцитів. Для якої анемії це притаманно?
A 58-year-old patient complains of increased fatigue, reduced work capacity, drowsiness, shortness of breath when walking fast. In the blood: er.-4.61012/l, Hb- 92 g/l, calc. readings.- 0.6. In the smear

blood is high the number of annulocytes and microcytes. For which anemia is this characteristic?

Перніціозна Pernicious

Серповидноклітинна Sickle cell

Гемолітична Hemolytic

Постгеморагічна Posthemorrhagic

Залізодефіцитна Iron deficiency

141 / 200
Хворий багато років страждав на бронхіальну астму та помер від нападу ядухи. Під час гістологічного дослідження легень виявлено: в просвіті бронхіол та дрібних бронхів багато слизу з домішкою еозинофілів, склероз міжальвеолярних перетинок, розширення просвіту альвеол. Який механізм розвитку реакції гіперчутливості мав місце? The patient suffered from bronchial asthma for many years and died from an attack of dysentery. During the histological examination of the lungs, it was found: in the lumen of the bronchioles and small bronchi there is a lot of mucus with an admixture of eosinophils, sclerosis interalveolar membranes, expansion of the lumen of the alveoli. What mechanism of the development of the hypersensitivity reaction took place?

Цитоліз, обумовлений лімфоцитами Cytolysis caused by lymphocytes

Гранулематоз Granulomatosis

Імунокомплексна реакція Immunocomplex reaction

Реагінова реакція Reagin reaction

Цитотоксична реакція Cytotoxic reaction

142 / 200
У хворого 53 років після поранення в ділянку промежини відмічається довільне сечовиділення. Який з м’язів найбільш вірогідно ушкоджений? A 53-year-old patient has involuntary urination after an injury in the perineum. Which muscle is most likely damaged?

M.ischiocavemosus M.ischiocavemosus

M.sphyncter uretrae M.sphyncter urethrae

M.transversus реrineі superficialis M.transversus rerinei superficialis

M.transversus реrіnеі profundus M.transversus rerinei profundus

M.bulbospongiosus M.bulbospongiosus

143 / 200
При обстеженні на бактеріоносійство черевного тифу у сироватці крові кухарки шкільної їдальні виявлені Vі-антитіла. Яка з перелічених реакцій була використана у данному випадку? Vi-antibodies were detected in the blood serum of the school canteen cook during the examination for typhoid bacteria. Which of the listed reactions was used in this case?

Реакція Відаля Vidal reaction

РІФ RIF

ІФА IFA

РЗК RZK

РНГА RNHA

144 / 200
Хвора надійшла до клініки зі скаргами на загальну слабкість, запаморочення, задишку. Незадовго до звернення у клініку вона приймала левоміцетин для профілактики кишкових інфекцій. У крові:

ер.- 1,91012/л, Hb- 58 г/л, колірний показник- 0,9; лейк.- 2,2109/л. Про яку анемію це свідчить?
The patient came to the clinic with with complaints of general weakness, dizziness, shortness of breath. Shortly before going to the clinic, she took chloramphenicol to prevent intestinal infections. In the blood:

er. - 1.91012/l, Hb - 58 g/l, color index - 0, 9; leuk.- 2.2109/l. What kind of anemia does this indicate?

Гемолітична Hemolytic

Метапластична Metaplastic

Апластична Aplastic

Гіпопластична Hypoplastic

Залізодефіцитна Iron deficiency

145 / 200
До клініки потрапила дитина 1 року з ознаками ураження м’язів. Після обстеження виявлений дефіцит карнитину у м’язах. Порушення якого процесу є біохімічною основою цієї патології? A 1-year-old child was admitted to the clinic with signs of muscle damage. After the examination, carnitine deficiency was detected in the muscles. The violation of which process is the biochemical basis of this pathology?

Синтез актину та міозину Synthesis of actin and myosin

Транспорт жирних кислот у мітохондрії Регуляція рівня Ca2+ B. в мітохондріях Transport of fatty acids in mitochondria Regulation of the level of Ca2+ B. in mitochondria

Утилізація молочної кислоти Disposal of lactic acid

Субстратне фосфорилювання Substrate Phosphorylation

146 / 200
Жінка 38 років скаржиться на підвищену пітливість, серцебиття, підвищення температури тіла у вечірні години. Основний обмін збільшений на 60%. Лікар встановив діагноз тиреотоксикоз. Які властивості тироксину приводять до підсилення теплопродукції? A 38-year-old woman complains of increased sweating, palpitations, increased body temperature in the evening hours. The basic metabolism is increased by 60%. The doctor diagnosed thyrotoxicosis. What properties of thyroxine lead to to increase heat production?

Зменшує в-окиснення жирних кислот Reduces in-oxidation of fatty acids

Зменшує дезамінування амінокислот Reduces amino acid deamination

Підвищує спряження окиснення та фосфорилування Increases the coupling of oxidation and phosphorylation

Сприяє накопиченню ацетил-КоА Contributes to the accumulation of acetyl-CoA

Розщеплює окисне фосфорилування Cleaves oxidative phosphorylation

147 / 200
У людини збережена смакова, але втрачена загальна чутливість структур ротової порожнини. Про ураження якого нерва це свідчить? A person has preserved taste, but lost the general sensitivity of the structures of the oral cavity. Which nerve does this indicate?

N.glossopharyngeus N.glossopharyngeus

N.trigemmus N.trigemmus

N.glossopharyngeus та n.vagus N.glossopharyngeus and n.vagus

N.hypoglossus N.hypoglossus

N.vagus N.vagus

148 / 200
До приймального відділення лікарні надійшов непритомний юнак з ознаками отруєння морфіном. Відзначається поверхневе та рідке дихання, яке обумовлене пригніченням дихального центру. Який тип недостатності дихання виник при цьому? An unconscious young man with signs of morphine poisoning was admitted to the reception department of the hospital. Shallow and shallow breathing is noted, which is due to depression of the respiratory center. What type of respiratory failure occurred?

Вентиляційна обструктивна Ventilation obstructive

Вентиляційна рестриктивна Ventilation restrictive

Перфузійна Perfusion

Дифузійна Diffuse

Вентиляційна дисрегуляторна Ventilation dysregulatory

149 / 200
У хворого з серцевою недостатністю виникла аритмія, під час якої частота скорочень передсердь була 70, а шлуночків - 35/хв. Порушення якої функції провідної системи серця спостерігалося у хворого? A patient with heart failure developed an arrhythmia, during which the frequency of contractions of the atria was 70, and the frequency of the ventricles - 35/min. Which function of the conduction system of the heart was impaired in the patient ?

Збудливість та провідність Excitability and Conductivity

Збудливість Excitability

Скоротливість Truncity

Провідність Conductivity

Автоматизм Automation

150 / 200
До лікаря звернувся пацієнт з приводу пожовтіння склер та шкіри. Під час обстеження не виявлено ознак енцефалопатії, холемічного та ахолічного синдромів. Яка жовтяниця розвинулася у цього пацієнта? A patient consulted a doctor about yellowing of the sclera and skin. During the examination, no signs of encephalopathy, cholemic and acholic syndromes were found. What kind of jaundice developed in this patient?

Гемолітична Hemolytic

Ензимопатична Enzymopathic

Механічна Mechanical

Ядерна Nuclear

Паренхіматозна Parenchymatous

151 / 200
Жінка 63 років приймала пірацетам для відновлення функції ЦНС після ішемічного інсульту. Стан хворої значно покращився. Який механізм дії даного препарату? A 63-year-old woman took piracetam to restore CNS function after an ischemic stroke. The patient's condition improved significantly. What is the mechanism of action of this drug?

Блокада моноамінооксидази Monoamine oxidase blockade

Блокада дофамін-гідроксилази Blockade of dopamine hydroxylase

Покращення метаболізму в ЦНС Improving metabolism in the central nervous system

Блокада холінацетази Cholinacetase blockade

Блокада катехол-о-метилтрансферази Catechol-o-methyltransferase blockade

152 / 200
У хворого на рентгенограмі легень виявлено затемнення. Під час діагностичної експресбіопсії лімфатичного вузла бронха виявлено: сирний некроз, навколо якого розташовані епітеліоїдні та лімфоїдні пласти з домішками багатоядерних гігантських клітин. Вкажіть причину лімфаденіту: The patient's X-ray of the lungs showed darkening. During the diagnostic express biopsy of the lymph node of the bronchus, it was found: cheesy necrosis, around which are located epithelioid and lymphoid layers with impurities of multinucleated giant cells. Specify the cause of lymphadenitis:

Метастази раку Cancer metastases

Пневмонія Pneumonia

Аденовірусна інфекція Adenovirus infection

Сифіліс Syphilis

Туберкульоз Tuberculosis

153 / 200
Хворий помер з явищами уремії. На розтині нирки збільшені, в’ялої консистенції, кіркова речовина широка, набрякла, з червоним крапом; мозкова речовина темно-червона. Мікроскопічно у порожнині капсули ниркового тільця виявлені “півмісяці' які стискають капіляри, дистрофія нефроцитів, набряк та інфільтрація строми. Яке захворювання стало причиною смерті цього чоловіка? The patient died with symptoms of uremia. At autopsy, the kidneys were enlarged, flaccid in consistency, the cortical substance was wide, swollen, with a red speck; the medullary substance was dark red. Microscopically in the cavity of the capsule of the renal corpuscle, 'crescents' were found that squeeze the capillaries, nephrocyte dystrophy, edema and stroma infiltration. What disease caused the death of this man?

Гломерулонефрит Glomerulonephritis

Нефротичний синдром Nephrotic syndrome

Нефролітіаз Nephrolithiasis

Амілоїдоз нирок Kidney amyloidosis

Пієлонефрит Pyelonephritis

154 / 200
У хворого струс головного мозку, що супроводжується повторним блюванням та задишкою. В артерiальнiй крові: рН- 7,62; PC02 - 40 мм рт.ст. Яке порушення кислотноосновного стану у хворого? The patient has a brain concussion, accompanied by repeated vomiting and shortness of breath. In arterial blood: pH - 7.62; PC02 - 40 mm Hg. What a violation acid-base condition in the patient?

Негазовий ацидоз Nongaseous acidosis

Газовий алкалоз Gas alkalosis

Негазовий алкалоз Nongaseous alkalosis

- -

Газовий ацидоз Gas acidosis

155 / 200
У хворого на жовчокам’яну хворобу розвинулася механічна жовтяниця. Під час обстеження встановлено, що камінь знаходиться у загальній жовчній протоці. Які жовчовивідні протоки утворюють обтуровану протоку? A patient with cholelithiasis developed mechanical jaundice. During the examination, it was established that the stone is in the common bile duct. Which bile ducts form an obstructed duct?

Ductus hepaticus dexter et smster Ductus hepaticus dexter et smster

Ductus hepaticus dexter et ductus cysticus Ductus hepaticus dexter et ductus cysticus

Ductus hepaticus commurns et ductus choledochus Ductus hepaticus commurns et ductus choledochus

Ductus hepaticus smster et ductus cysticus Ductus hepaticus smster et ductus cysticus

Ductus hepaticus commurns et ductus cysticus Ductus hepaticus commurns et ductus cysticus

156 / 200
У дитини першого року життя під час профілактичного огляду виявлено порушення мінералізації кісток. Нестача якого вітаміну може бути причиною цього? A child of the first year of life during a preventive examination was found to have a bone mineralization disorder. A lack of which vitamin can be the cause of this?

Токоферол Tocopherol

Кальциферол Calciferol

Фолієва кислота Folic acid

Рибофлавін Riboflavin

Кобаламін Cobalamin

157 / 200
У чоловіка 30 років методом непрямої калориметрії встановлено, що його основний обмін на 30% менше від належного. Знижена секреція гормонів якої залози (яких залоз) є причиною цього? In a 30-year-old man, it was determined by the method of indirect calorimetry that his basic metabolism is 30% less than it should be. Decreased hormone secretion of which gland(s) is the reason for this?'

Наднирники Adrenal glands

Епіфіз Pineal gland

Щитовидна Thyroid

Підшлункова Pancreatic

Прищитоподібні Pystiliformes

158 / 200
Новонароджена дитина має недорозвинений тимус. Який вид гемопоезу буде порушений? A newborn child has an underdeveloped thymus. What type of hematopoiesis will be impaired?

Мегакаріоцитопоез Megakaryocytopoiesis

Моноцитопоез Monocytopoiesis

Лімфопоез Lymphopoiesis

Гранулоцитопоез Granulocytopoiesis

Еритропоез Erythropoiesis

159 / 200
На мікропрепараті серця розрізняються клітини прямокутної форми, розмірами від 50 до 120 мкм, з центрально розташованим ядром, розвиненими міофібрилами, зв’язані між собою вставними дисками. Яку функцію виконують ці клітини? On the micro-section of the heart, cells of a rectangular shape, sizes from 50 to 120 μm, with a centrally located nucleus, developed myofibrils, connected to each other by intercalated discs, are distinguished. What is the function of do these cells?

Проведення імпульсів Pulsing

Скорочення серця Shortening of the heart

Регенераторна Regenerative

Ендокринна Endocrine

Захисна Protective

160 / 200
У збудливій клітині заблокували іонні канали. Це суттєво не змінило рівень потенціалу спокою, але клітина втратила здатність до генерації ПД. Які канали заблоковано? Ion channels were blocked in an excitable cell. This did not significantly change the level of the resting potential, but the cell lost the ability to generate PD. Which channels are blocked?

Натрієві та калієві Sodium and potassium

Натрієві Sodium

Хлорні Chlorni

Кальцієві Calcium

Калієві Potassium

161 / 200
У людини крововилив у задню центральну звивину призвів до порушення чутливості з протилежного боку. Який вид чутливості порушений? In a person, a hemorrhage in the posterior central gyrus led to impaired sensitivity on the opposite side. What type of sensitivity is impaired?

Зорова Zorova

Нюхова та смакова Olfactory and gustatory

Слухова та зорова Aural and visual

Шкірна та пропріоцептивна Skin and proprioceptive

Слухова Auditory

162 / 200
У пацієнта розвинулися порушення рухової активності: тремор, атаксія та асинергія рухів, дизартрія. Яка структура найбільш вірогідно уражена? The patient developed disturbances of motor activity: tremor, ataxia and asynergy of movements, dysarthria. Which structure is most likely affected?

Базальні ганглії Basal ganglia

Стовбур мозку Brain stem

Мозочок Cerebellum

Довгастий мозок Olongate brain

Лімбічна система Limbic system

163 / 200
Хворому на миготливу аритмію, в анамнезі якого бронхіальна астма, треба призначити протиаритмічний засіб. Який препарат з цієї групи ПРОТИПОКАЗАНИЙ хворому? A patient with atrial fibrillation with a history of bronchial asthma should be prescribed an antiarrhythmic drug. Which drug from this group is CONTRAINDICATED for the patient?

Ніфедипін Nifedipine

Аймалін Aimalin

Новокаїнамід Novocaineamide

Верапаміл Verapamil

Анаприлін Anaprilin

164 / 200
На електронній мікрофотографії представлена клітина нейрального походження, що знаходиться у складі епітелію слизової оболонки. Дистальна частина периферійного відростку клітини має булавоподібне потовщення, від якого відходять 10-12 війок. Що це за клітина? The electron micrograph shows a cell of neural origin, which is part of the epithelium of the mucous membrane. The distal part of the peripheral process of the cell has a mace-like thickening, from which 10-12 cilia depart. What kind of cell is this?

Палочкова зорова клітина Rod optic cell

Сенсорна клітина смакової цибулини Taste bulb sensory cell

Біполярний нейрон спинномозкового вузла Bipolar neuron of the spinal cord

Колбочкова зорова клітина Cone eye cell

Нюхова Nyukhova

165 / 200
Ізольована клітина серця людини автоматично генерує імпульси збудження з частотою 60 разів за хвилину. З якої структури серця отримано цю клітину? An isolated human heart cell automatically generates excitation pulses with a frequency of 60 times per minute. From which heart structure is this cell derived?

Пучок Гіса His Bundle

Шлуночок Venticle

Синоатриальний вузол Sinoatrial node

Атріовентрикулярний вузол Atrioventricular node

Передсердя Atrial

166 / 200
У людини через 10 хвилин після початку інтенсивної фізичної роботи кількість еритроцитів у крові збільшилася з 4,01012/л до 4,51012/л. Що є основною причиною цього? A person has 10 minutes after the beginning of intensive physical work, the number of erythrocytes in the blood increased from 4.01012/l to 4.51012/l. What is the main reason for this?

Збільшення хвилинного об’єму крові Increase in minute blood volume

Вихід еритроцитів з депо Output of erythrocytes from the depot

Активація еритропоезу Activation of erythropoiesis

Пригнічення руйнування еритроцитів Inhibition of erythrocyte destruction

Втрата води організмом Body water loss

167 / 200
У людини в артеріальній крові напруга кисню збільшена до 104 мм рт.ст., а вуглекислого газу зменшена до 36 мм рт.ст. Що може бути причиною цього? In a person, the tension of oxygen in the arterial blood is increased to 104 mm Hg, and carbon dioxide is reduced to 36 mm Hg. What could be the reason for this?

Довільна гіпервентиляція Arbitrary Hyperventilation

Інтенсивне фізичне навантаження Intensive physical activity

Перебування у горах Staying in the mountains

Помірне фізичне навантаження Moderate physical activity

Затримка дихання Respiration delay

168 / 200
У чоловіка 33 років діагностовано прободіння шлунка та запалення очеревини, що призвело до напруження м’язів передньої черевної стінки (“доскоподібний живіт”). Який рефлекс забезпечує цей симптом? A 33-year-old man was diagnosed with a perforation of the stomach and inflammation of the peritoneum, which led to the tension of the muscles of the anterior abdominal wall ('plank-like stomach'). What reflex provides this symptom ?

Вісцеро-кутанний Viscerocutaneous

Вісцеро-соматичний Viscero-somatic

Вісцеро-вісцеральний Viscero-visceral

Сомато-вісцеральний Somato-visceral

Кутанно-вісцеральний Cutaneous-visceral

169 / 200
У хворого глибока рвана рана із нерівними краями, вкрита гноєм. У крайових відділах сочна грануляційна тканина, яка не здіймається над рівнем рани. Назвіть вид загоєння рани: The patient has a deep torn wound with uneven edges, covered with pus. In the marginal areas, there is juicy granulation tissue that does not rise above the level of the wound. Name the type of wound healing:

Безпосереднє закриття дефекту епітеліальної тканини Direct closure of epithelial tissue defect

Під струпом Under the scab

Первинним натягом Primary tension

- -

Вторинним натягом Secondary tension

170 / 200
Під час гістологічного дослідження лімфовузла хворого 18 років з ділянки заднього трикутника шиї морфолог виявив скопичення клітин, серед яких поодинокі багатоядерні клітини Березовського-Штернберга, великі клітини Ходжкіна, малі клітини Ходжкіна та багато лімфоцитів, поодинокі плазматичні клітини, еозинофіли. Яке захворювання у хворого? During the histological examination of the lymph node of an 18-year-old patient from the posterior triangle of the neck, the morphologist found an accumulation of cells, including single Berezovsky-Sternberg multinucleated cells, large Hodgkin cells, small cells Hodgkin's disease and many lymphocytes, single plasma cells, eosinophils. What disease does the patient have?

Хронічний лімфоїдний лейкоз Chronic lymphoid leukemia

Пухлина Беркіта Burkitt's tumor

Лімфогранулематоз Lymphogranulomatosis

Лімфоцитарна лімфома Lymphocytic lymphoma

Нодулярна лімфома Nodular lymphoma

171 / 200
Дівчинка 6 років захворіла на дифтерію та померла від асфіксії на третю добу. На аутопсії слизова оболонка трахеї та бронхів стовщена, набрякла, тьмяна, вкрита сіруватими плівками, які легко відокремлюються. Про який вид запалення свідчать морфологічні зміни? A 6-year-old girl fell ill with diphtheria and died of asphyxiation on the third day. At autopsy, the mucous membrane of the trachea and bronchi is thickened, swollen, dull, covered with grayish films that are easily are separated. What kind of inflammation do the morphological changes indicate?

Дифтеритичне Diphtheritic

Серозне Serious

Крупозне True

Катаральне Catarrhal

Геморагічне Hemorrhagic

172 / 200
Хворому з травмою передпліччя під час репозиції кісток для міорелаксації введено дитилін. Повне відновлення тонусу та функції м’язів відбулося більш, ніж через годину. Чим можна пояснити значне подовження курареподібної дії препарату? Dithyline was administered to a patient with a forearm injury during bone repositioning for myorelaxation. Full recovery of muscle tone and function occurred in more than an hour. What can explain the significant lengthening curare-like effect of the drug?

Пригнічєнням мікросомного окиснення Inhibition of microsomal oxidation

Генетичним дефіцитом гідроксилази Genetic hydroxylase deficiency

Генетичним дефіцитом бутирилхолінестерази Genetic deficiency of butyrylcholinesterase

Генетичним дєфіцитом моноамінооксидази Genetic deficiency of monoamine oxidase

Утворенням активного метаболіту Formation of an active metabolite

173 / 200
У людини, що виконувала важку фізичну роботу в умовах підвищеної температури навколишнього середовища, змінилася кількість білків плазми крові. Що саме має місце у даному випадку? The amount of blood plasma proteins has changed in a person who performed heavy physical work in conditions of increased ambient temperature. What exactly is happening in this case?

Парапротеїнемія Paraproteinemia

Відносна гіперпротеїнемія Relative hyperproteinemia

Абсолютна гіпопротеїнемія Absolute hypoproteinemia

Абсолютна гіпепротеїнемія Absolute hyperproteinemia

Диспротеїнемія Dysproteinemia

174 / 200
Хворому, що страждає на стенокардію та приймає ізосорбіда мононітрат, було додатково призначено лікарський засіб з дезагрегантним ефектом. Визначте цей препарат: A patient suffering from angina and taking isosorbide mononitrate was additionally prescribed a drug with a disaggregant effect. Identify this drug:

Ніфедипін Nifedipine

Анаприлін Anaprilin

Нітрогліцерин Nitroglycerin

Ацетилсаліцилова кислота Acetylsalicylic acid

Валідол Validol

175 / 200
Хворому в післяопераційному періоді для стимуляції перистальтики кишечника та тонусу сечового міхура було призначено препарат з групи антихолінестеразних засобів. Визначте його серед нижченаведених препаратів: A drug from the group of anticholinesterase drugs was prescribed to the patient in the postoperative period to stimulate intestinal peristalsis and bladder tone. Identify it among the following drugs:

Дихлотіазид Dichlorothiazide

Анаприлін Anaprilin

Резерпін Reserpin

Прозерин Proserin

Маніт Manit

176 / 200
У хворого діагностовано ураження стулок правого передсердно-шлуночкового клапана. Внаслідок запального процесу якої анатомічної структури серця відбулися зміни у стулках? The patient was diagnosed with damage to the leaflets of the right atrioventricular valve. As a result of the inflammatory process, which anatomical structure of the heart underwent changes in the leaflets?

Серозний перикард Serous pericardium

Епікард Epicard

Міокард Myocardium

Ендокард Endocard

Фіброзний перикард Fibrous pericardium

177 / 200
У хворого на обличчі вугрі. Під час мікроскопії зскрібків із уражених ділянок виявлені живі членистоногі розміром 0,2-0,5 мм., які мають витягнуту червоподібну форму, чотири пари коротких кінцівок, що розташовані у середній частині тіла. Який лабораторний діагноз? The patient has acne on his face. During the microscopy of scrapings from the affected areas, living arthropods with a size of 0.2-0.5 mm, which have an elongated worm-like shape, were found. four pairs of short limbs located in the middle part of the body. What is the laboratory diagnosis?

Педикульоз Pediculosis

Демодекоз Demodecosis

Міаз Miaz

Фтиріоз Phthyriosis

Короста Scabies

178 / 200
У хворого, який тривалий час страждає на хронічний ентероколіт, після вживання молока виникли метеоризм, діарея, коліки. З нестачею якого ферменту в кишечнику це пов’язано? A patient who has been suffering from chronic enterocolitis for a long time developed flatulence, diarrhea, and colic after drinking milk. This is due to the lack of which enzyme in the intestines?

Лактаза Lactase

Сахараза Sucrase

Мальтаза Maltase

Глікогенситнетаза Glycogen synthetase

Амілаза Amylase

179 / 200
У новонародженної дитини виявлено гнійні виділення з кон’юктиви ока. Під час мікроскопії мазка з кон’юктиви знайдено велику кількість лейкоцитів та грамнегативні бобоподібні диплококи, що знаходяться всередині лейкоцитів. Який збудник є причиною цього захворювання? Purulent discharge from the conjunctiva of the eye was detected in a newborn child. During the microscopy of the smear from the conjunctiva, a large number of leukocytes and gram-negative bean-shaped diplococci were found inside the leukocytes What pathogen is the cause of this disease?

Staphylococcus epidermidis Staphylococcus epidermidis

Neisseria gonorrhoeae Neisseria gonorrhoeae

Streptococcus pyogenes Streptococcus pyogenes

Neisseria catarrhalis Neisseria catarrhalis

Staphylococcus aureus Staphylococcus aureus

180 / 200
Жінці 30 років на 32 тижні вагітності лікар, в складі комплексної терапії виразкової хвороби шлунка, призначив антибіотик. Вкажіть, який препарат можна застосувати у даному випадку? To a 30-year-old woman at 32 weeks of pregnancy, the doctor prescribed an antibiotic as part of complex therapy for peptic ulcer disease. Specify which drug can be used in this case?

Азитроміцин Azithromycin

Тетрациклін Tetracycline

Бензилпеніцилін Benzylpenicillin

Гентаміцин Gentamicin

Левоміцетин Levomycetin

181 / 200
Під час ультразвукового обстеження вагітної жінки було діагностовано багатоводдя. З порушенням діяльності яких позазародкових органів можна пов’язати даний патологічний стан? During the ultrasound examination of a pregnant woman, polyhydramnios was diagnosed. This pathological condition can be associated with a violation of the activity of which extraembryonic organs?

Жовтковий мішок Yolk Sac

Хоріон Chorio

Плацента Placenta

Амніотична оболонка Amniotic membrane

Алантоїс Allantois

182 / 200
У хворого з гострим циститом під час дослідження сечі виявили лейкоцити та багато грамнегативних паличок. При посіві виросли колонії слизового характеру, які утворювали зелений розчинний пігмент. Який мікроорганізм, найбільш вірогідно, є причиною захворювання? In a patient with acute cystitis, leukocytes and many gram-negative rods were found during the urine examination. When cultured, colonies of a mucous nature grew, which formed a green soluble pigment. What microorganism, the most is probably the cause of the disease?

Pseudomonas aeruginosa Pseudomonas aeruginosa

Salmonella enteritidis Salmonella enteritidis

Escherihia coli Escherichia coli

Klebsiella pneumoniae Klebsiella pneumoniae

Proteus mirabilis Proteus mirabilis

183 / 200
Чоловіка 49 років доставлено з місця автомобільної аварії до лікарні у непритомному стані. Шкірні покриви бліді, пульс частий та поверхневий. Переломів кісток та пошкодження головного мозку не виявлено. Під час пункції черевної порожнини отримано значну кількість крові. Що є первинною причиною важкого стану потерпілого? A 49-year-old man was brought from the scene of a car accident to the hospital in an unconscious state. The skin is pale, the pulse is frequent and superficial. No bone fractures and brain damage were found. Under during the puncture of the abdominal cavity, a significant amount of blood was obtained. What is the primary cause of the serious condition of the victim?

Гіпоінсулінемія Hyposulinemia

Еритропенія Erythropenia

Гіпопротеїнемія Hypoproteinemia

Гіповолемія Hypovolemia

Гіпонатріємія Hyponatremia

184 / 200
У жінки 45 років народився хлопчик з розщепленням верхньої щелепи (“заяча губа” та “вовча паща”). Під час додаткового обстеження виявлені значні порушення з боку нервової, серцево-судинної систем та зору. При дослідженні каріотипу діагностована трісомія по 13 хромосомі. Який синдром має місце у хлопчика? A 45-year-old woman gave birth to a boy with a split upper jaw ('hare's lip' and 'wolf's mouth'). During an additional examination, significant disturbances from the nervous, of the cardiovascular system and vision. Trisomy on chromosome 13 was diagnosed during the karyotype study. What syndrome does the boy have?

Патау Patau

Дауна Down

Едвардса Edwards

Клайнфельтера Klinefelter

Шерешевського-Тернера Shereshevsky-Turner

185 / 200
Під час медико-генетичного консультування родини зі спадковою патологією виявлено, що аномалія проявляється через покоління у чоловіків. Який тип успадковування притаманний для цієї спадкової аномалії? During medical and genetic counseling of a family with a hereditary pathology, it was found that the anomaly manifests itself through generations in men. What type of inheritance is inherent in this hereditary anomaly?

Х-зчеплене домінантне X-linked dominant

Х-зчеплене рецесивне X-linked recessive

Аутосомне рецесивне Autosomal recessive

Y-зчеплене Y-linked

Аутосомне домінантне Autosomal dominant

186 / 200
Людина 28 років споживає надмірну кількість вуглеводів (600 г на добу), що перевищує її енергетичні потреби. Який процес буде активуватися у даному випадку? A 28-year-old man consumes an excessive amount of carbohydrates (600 g per day), which exceeds his energy needs. What process will be activated in this case?

Ліпогенез Lipogenesis

Ліполіз Lipolysis

Гліколіз Glycolysis

Глюконеогенез Gluconeogenesis

Окислення жирних кислот Oxidation of fatty acids

187 / 200
У дитини 5 років діагностовано хворобу Брутона, яка проявляється у важкому перебігу бактеріальних інфекцій, відсутності В-лімфоцитів та плазматичних клітин. Які зміни вмісту імуноглобулінів будуть спостерігатися у сироватці крові цієї дитини? A 5-year-old child was diagnosed with Bruton's disease, which manifests itself in a severe course of bacterial infections, the absence of B-lymphocytes and plasma cells. What changes in the content of immunoglobulins will be observed in the blood serum of this child?

Збільшення IgD,IgE Increase of IgD,IgE

Збільшення IgA, IgM IgA, IgM increase

Зменшення IgA, IgM Reduction of IgA, IgM

Змін не буде There will be no changes

Зменшення IgD,IgE Reduction of IgD,IgE

188 / 200
У жінки 30 років виявлено недостатність зовнішньосекреторної функції підшлункової залози. Гідроліз яких поживних речовин буде порушений? A 30-year-old woman was found to have insufficiency of the exocrine function of the pancreas. The hydrolysis of which nutrients will be impaired?

Білки, жири Proteins, fats

Білки, жири, вуглеводи Proteins, fats, carbohydrates

Жири, вуглеводи Fats, carbohydrates

Білки, вуглеводи Proteins, carbohydrates

Білки Proteins

189 / 200
На практичному занятті з мікробіології студентам запропоновано пофарбувати готові зафіксовані мазки із мокротиння хворого на туберкульоз. Який метод фарбування треба використати у данному випадку? In a practical lesson in microbiology, students were asked to stain ready-made fixed smears from the sputum of a tuberculosis patient. What staining method should be used in this case?

Буррі Burri

Романовського-Гімза Romanovsky-Giems

Грама Gram

Циля-Нільсена Nielsen Target

Гінса Guinsa

190 / 200
Під час обстеження на бактеріоносійство працівників дитячих закладів у виховательки виділена С.diphtheriae. Було проведено дослідження на токсигенність збудника, яке показало, що цей штам С.diphtheriae не продукує екзотоксин. Яку реакцію провели при дослідженні на токсигенність дифтерійних бактерій? During the bacteriological examination of employees of children's institutions, S. diphtheriae was isolated from the teacher. A study was conducted on the toxigenicity of the pathogen, which showed that this strain of S. diphtheriae does not produce exotoxin. What reaction was carried out during the study on the toxigenicity of diphtheria bacteria?

Реакція імунофлуоресценсії Immunofluorescence reaction

Реакція кільцепреципітації Ring precipitation reaction

Реакція зв’язування комплементу Complement binding reaction

Реакція аглютинації Agglutination reaction

Реакція преципітації в агаровому гелі Agar gel precipitation reaction

191 / 200
У хворої параліч мімічної мускулатури, розлад сприйняття смаку в передніх 2/3 язика, зменшення слиновиділення. Який з черепних нервів уражений? The patient has paralysis of the mimic muscles, a disorder of taste perception in the front 2/3 of the tongue, decreased salivation. Which of the cranial nerves is affected?

N.glossphapyngeus N.glossphapyngeus

N.trigeminus N.trigeminus

N.hypoglossus N.hypoglossus

N.facialis N.facialis

N.vagus N.vagus

192 / 200
Лікар-дослідник у складі альпіністської експедиції піднявся у базовий табір, розташований на висоті 5000 м. На 3-й день перебування у нього з’явилися ознаки гірської хвороби: задишка, головний біль, втрата апетиту, загальна слабкість, ціаноз. Який тип гіпоксії має місце в цьому випадку? A research doctor as part of a mountaineering expedition climbed to the base camp, located at an altitude of 5000 m. On the 3rd day of his stay, he developed symptoms of mountain sickness: shortness of breath, headache, loss of appetite, general weakness, cyanosis. What type of hypoxia occurs in this case?

Циркуляторна Circulator

Змішана Mixed

Тканинна Fabric

Гіпоксична Hypoxic

Гемічна Chemical

193 / 200
У хворого на колагеноз після тривалого прийому преднізолону з’явився спастичний біль скелетних м’язів внаслідок розвитку гипокаліємії. Який препарат треба використати для корекції обміну калію? A patient with collagenosis developed spastic pain in skeletal muscles due to the development of hypokalemia after long-term use of prednisone. What drug should be used to correct potassium metabolism?

Діазепам Diazepam

Тирокальцитонін Tyrocalcitonin

Но-шпа No-shpa

Дитилін Ditylin

Панангін Panangin

194 / 200
У робітника 37 років, який працював у кесоні, після підйому на поверхню раптово з’явилися ознаки гострого порушення мозкового кровообігу, непритомність. Через декілька днів він помер. На розтині в лівій півкулі головного мозку виявлено вогнище сірого кольору м’якої консистенції, неправильної форми, розмірами 5х6х3,5 см. Який процес мав місце у головному мозку? A 37-year-old worker who worked in a caisson, after rising to the surface, suddenly showed signs of an acute violation of cerebral blood circulation, fainting. He died a few days later. the autopsy revealed a gray focus of soft consistency, irregular shape, dimensions 5x6x3.5 cm in the left hemisphere of the brain. What process took place in the brain?

Абсцес Abscess

Ішемічний інсульт Ischemic stroke

Геморагічний інсульт Hemorrhagic stroke

Кіста Cyst

Пухлина Tumor

195 / 200
Зі слизових оболонок та з харкотиння хворого, який тривалий час приймав імунодепресанти, були виділені грампозитивні великі овальні клітини з брунькуванням, що розташовані хаотично, та подовжені клітини у вигляді ланцюжків. Який збудник виділений? From the mucous membranes and sputum of a patient who took immunosuppressants for a long time, gram-positive large oval cells with budding, randomly located, and elongated cells in the form of chains were isolated What pathogen is isolated?

Стрептококи Streptococci

Ієрсінії Yersinia

Актиноміцети Actinomycetes

Стрептобактерії Streptobacteria

Кандиди Candida

196 / 200
У здорової дорослої людини проводять зондування порожнин серця. Зонд знаходится у лівому шлуночку. Під час якої фази (періоду) серцевого циклу буде зареєстровано збільшення тиску від 8 до 70 мм рт.ст.? In a healthy adult, the heart cavities are probed. The probe is in the left ventricle. During which phase (period) of the cardiac cycle will an increase in pressure from 8 to 70 mm be registered Hg?

Фаза швидкого вигнання Quick Expulsion Phase

Фаза повільного вигнання Slow Expulsion Phase

Фаза ізометричного скорочення Isometric contraction phase

Період вигнання Exile Period

Фаза асинхронного скорочення Asynchronous reduction phase

197 / 200
Людина за призначенням лікаря тривалий час приймала препарат з групи глюкокортикоїдних гормонів. Секреція якого (яких) з наведених гормонів буде пригнічена внаслідок цього? A person has been taking a drug from the group of glucocorticoid hormones for a long time as prescribed by a doctor. The secretion of which (which) of these hormones will be suppressed as a result?

Кортикотропний Corticotropic

Статеві Gender

Мінералокортикоїди Mineralocorticoids

Тиротропний Thyrotropic

Соматотропний Somatotropic

198 / 200
У студента через 2 години після іспиту в аналізі крові виявлено лейкоцитоз без істотних змін у лейкоцитарній формулі. Який найбільш вірогідний механізм розвитку лейкоцитозу? Two hours after the exam, the student's blood test revealed leukocytosis without significant changes in the leukocyte formula. What is the most likely mechanism of leukocytosis?

Посилення лейкопоезу та зменшення руйнування лейкоцитів Increasing leukopoiesis and reducing destruction of leukocytes

Перерозподіл лейкоцитів в організмі Redistribution of leukocytes in the body

Уповільнення руйнування лейкоцитів Slowing down the destruction of leukocytes

Посилення лейкопоезу Increasing leukopoiesis

Уповільнення міграції лейкоцитів у тканини Slowing down the migration of leukocytes into tissues

199 / 200
У молодої жінки видалили пухлину дистального кінця стегнової кістки, яка швидко зростала. Макроскопічно: пухлина строкатого вигляду - від біло-сірого до коричневочервоного кольору, пухкої консистенції. Мікроскопічно: основний тканинний компонент пухлини представлений кістковими та остеоїдними структурами, вистеленими атиповими остеобластами з патологічними мітозами; безліч тонкостінних судин. Який найбільш вірогідний діагноз? A tumor of the distal end of the femur, which was growing rapidly, was removed from a young woman. Macroscopically: a tumor of a variegated appearance - from white-gray to brownish-red in color, loose consistency. Microscopically: the main tissue component of the tumor is represented by bone and osteoid structures lined by atypical osteoblasts with pathological mitoses; many thin-walled vessels. What is the most likely diagnosis?

Остеома Osteoma

Саркома Юїнга Ewing's sarcoma

Ангіосаркома Angiosarcoma

Хондрома Chondroma

Остеосаркома Osteosarcoma

200 / 200
У хворого підвищений вміст глюкози в крові; є цукор в сeчі. Під час пункційної біопсії нирки виявлено: розширення мезангія з осередковим накопиченням мембраноподібної речовини з перигломерулярним склерозом деяких клубочків, гіаліноз та плазматичне просякання артеріол, лімфогістіоцитарна інфільтрація строми з наявністю поліморфноядерних лейкоцитів; глікогенна інфільтрація нефроцитів вузького сегмента. Який найбільш вірогідний діагноз? The patient has an elevated blood glucose level; there is sugar in the urine. During a puncture biopsy of the kidney, it was found: expansion of the mesangium with focal accumulation of a membrane-like substance with periglomerular sclerosis of some glomeruli, hyalinosis and plasmatic percolation of arterioles, lymphohistiocytic infiltration of the stroma with the presence of polymorphonuclear leukocytes, glycogenic infiltration of nephrocytes of a narrow segment. What is the most likely diagnosis?

Діабетичний гломерулосклероз Diabetic glomerulosclerosis

Пієлонефрит Pyelonephritis

Гострий гломерулонефрит Acute glomerulonephritis

Хронічний гломерулонефрит Chronic glomerulonephritis

Підгострий гломерулонефрит Subacute glomerulonephritis